Sie sind auf Seite 1von 32

Egyptian_Pediatric yahoo group

Egyptian_Pediatric yahoo group

http://health.groups.yahoo.com/group/ egyptian_pediatric/

http://health.groups.yahoo.com/group/ egyptian_pediatric/

SELF-ASSESSMENT

Self-assessment
Questions
Question 1 A 3-year-old boy presents with septic shock secondary to meningococcal septicaemia, and is treated in the regional paediatric intensive care unit (PICU). He is treated with 7 days of intravenous (i.v.) ceftriaxone 50 mg/kg daily. He required 180 ml/kg of uid in the rst 24 hours, received adrenaline and milrinone infusions until day 3, but was successfully extubated on day 4, and returned to the ward on day 5. He has not had a lumbar puncture. It is now day 7, and his antibiotics are due to be discontinued. His mother reports he is less well in the past 24 hours he is miserable and does not want to get out of bed. He says his leg is sore. The discharge summary from PICU shows that he had been afebrile since day 2. He had his urinary catheter and central venous line removed on the day of discharge form PICU. On examination he has a temperature of 38.8 C. He has a few maculopapular spots on his trunk and upper arms, and two of them appear blistered. There are no petechiae. The rash is new. His chest is clear, his abdomen soft and his heart sounds normal. He has no neck stiffness. He has a slightly swollen right knee, with slight restriction of exion due to pain. His blood results from this morning are as follows:
Hb WBC Neutrophils Lymphocytes Platelets CRP 12 g/dl 22 109/l 17 109/l 4 109/l 432 109/l 89 mg/l (31 previous day) Na K Urea Creatinine 139 mmol/l 3.2 mmol/l 5.3 mmol/l 29 mol/l

paraldehyde. The seizure stopped as he was given thiopentone to induce anaesthesia. The total duration of seizure time was 55 minutes. He is ventilated with pressures of 22/6cm H2O at 25 per minute and 40% oxygen giving a tidal volume of 90 ml, oxygen saturations of 100%. He has an area of right upper zone consolidation on his chest x-ray. His heart rate is 125/min, blood pressure 90/45mm Hg, mean 60mm Hg, with a capillary rell time of 2 seconds. He has required three 20 ml/kg boluses of 0.9% saline in the past 4 hours and has not been started on inotropes. His i.v. maintenance uid is running at 100% of normal requirements. He had a computed tomography (CT) of his head with contrast, which showed a degree of meningeal enhancement, but is otherwise normal. He is sedated with morphine and midazolam infusions. He has been treated with aciclovir and ceftriaxone. Blood results are shown:
Hb WBC Neutrophils Lymphocytes Platelets CRP 11.1 g/dl 16.4 109/l 11.9 109/l 3.1 109/l 194 109/l 142 mg/l Na K Urea Creatinine Arterial gas pH pCO2 pO2 HCO3 Base excess Chloride Lactate 139 mmol/l 3.7 mmol/l 2.6 mmol/l 22 mol/l

7.26 6.9 kPa 18.4 kPa 21 mmol/l 8.1 mmol/l 112 mmol/l 5.3 mmol/l

(a)  What is the most likely diagnosis? Chose ONE of the following: Drug reaction Recurrent meningococcal septicaemia Septic arthritis Post-meningococcal immune complex mediated disease Incompletely treated meningitis Question 2 A 1-year-old boy weighing 11 kg has been transferred to PICU after presenting to his local hospital with a 48-hour history of lethargy, poor feeding and fever, culminating in a prolonged tonic seizure. He needed to be intubated and ventilated during the seizure after being treated with lorazepam, phenytoin and

(a)  Which of the following statements is true? Choose ONE of the following: Lumbar puncture should be performed now His ventilator settings should be changed Inotropes should be started His i.v. uid requirements should be restricted He should have a bolus of 20 ml/kg 4% human albumin He remains relatively stable for the next 48 hours. Blood culture results have been telephoned through from his original hospital, and show Streptococcus pneumoniae. His observation chart shows stable heart rate 120130/min, blood pressure mean of 60 mmHg, stable and normal blood gases, urine output of 0.7 ml/kg/hour. His routine bloods for the day come back:
Hb WBC Neutrophils Lymphocytes Platelets CRP 7.6 g/dl 12.4 109/l 8.5 109/l 2.9 109/l 44 109/l 98 mg/l Na K Urea Creatinine Phosphate Calcium (ionised) Magnesium 129 mmol/l 4.3 mmol/l 14.5 mmol/l 147 mol/l 2.9 mmol/l 1.03 mmol/l 0.81 mmol/l

Allan Wardhaugh MB ChB MRCPCH MRCGP DRCOG is a Consultant in Paediatric Intensive Care at University Hospital of Wales Cardiff, UK.

PAEDIATRICS AND CHILD HEALTH 19:1

50

2008 Elsevier Ltd. All rights reserved.

SELF-ASSESSMENT

PT APTT Fibrinogen

12 s 31 s 2.1 g/l

(b)  What investigation should you do now? Choose ONE of the following: Stool culture Ferritin Coombs test Blood lm Renal ultrasound (c)  A provisional blood culture result is phoned through the following day and reports Gram-positive diplococci. What is the likely diagnosis? Choose ONE of the following: Haemolytic uraemic syndrome pneumococcal Disseminated intravascular coagulation Haemolytic uraemic syndrome Escherichia coli H7:0157 Idiopathic thrombocytopenia Primary bone marrow failure Question 3 An 11-year-old boy presents with a swollen left ankle. He had presented to Accident and Emergency 3 days ago after having hurt his ankle playing football. An x-ray at that time showed no fracture, and he was sent home. Since then his ankle has remained swollen and red, but his parents think it is no worse than at onset. The previous day he developed diarrhoea. He has also complained of aching arms and legs. A repeat x-ray does not demonstrate any fracture, and shows only soft tissue swelling. You have been asked to give a paediatric opinion. His past medical history is unremarkable, and there is no family history of note. On examination his temperature is 39.1 C, and he appears ushed. His heart rate is 120/minute, his capillary rell time is 2 seconds and his blood pressure is 84/38mm Hg. His respiratory rate is 24/minute. There is a 2 cm abrasion just above his medial malleolus, and his ankle is a little swollen and warm. It is painful to move, but he does have a full range of movement in the joint. You suspect he has cellulitis, and admit him to the ward for intravenous penicillin and ucloxacillin after taking blood for culture, full blood count, urea and electrolytes, and C-reactive protein (CRP). Over the next 12 hours he becomes more unwell. Nursing staff report him as being intermittently confused. His heart rate remains 120 per minute, and his systolic blood pressure is between 70 and 80 mmHg. He has developed a widespread erythematous rash which looks like sunburn, and has a bilateral conjunctivitis. His urine output is 0.3 ml/kg/hour. His laboratory results from admission are below:
Hb WBC Neutrophils Lymphocytes Platelets CRP 9.8 g/dl 4.8 109/l 1.0 109/l 3.2 109/l 44 109/l 256 mg/l Na K Urea Creatinine 131 mmol/l 4.2 mmol/l 10.2 mmol/l 182 mol/l

(a)  Which of the following diagnoses is most likely? Choose ONE of the following: Septic arthritis Kawasaki disease StevensJohnson syndrome Necrotising fasciitis Toxic shock syndrome (b)  In addition, discussing him with an intensivist, and giving him a 20 ml/kg bolus of uid, which one of the following would you do? Choose ONE of the following: Treat with intravenous clindamycin Treat with intravenous immunoglobulin Start high dose corticosteroids Obtain an ultrasound of his ankle Obtain a paediatric nephrology opinion to arrange early dialysis

Answers
Question 1 (a)  Post-meningococcal immune complex mediated disease In meningococcal disease, type 3 hypersensitivity reaction, or immune complex associated complications (IAC), can present as arthritis, vasculitis, episcleritis, or pericarditis. Nephritis is very rare. It usually takes 410 days after the onset of disease for the rst symptoms and signs of IAC to develop.1 The diagnostic features are:
Arthritis Arthralgia, joint swelling and redness, limitation of movement Synovial uid shows no bacteria, cultures remain negative Pustular, bullous, nodular lesions or rash Retrosternal pain, pericardial friction rub, ECG abnormalities, cardiac enlargement on x-ray/ultrasound Pleural effusion on x-ray. Pain on inspiration. Impaired percussion. Pleural rub

Vasculitis Pericarditis Pleuritis

Up to 15% of children may show one or more of these features after severe meningococcal disease. There is typically a secondary fever (i.e. a fever occurs after the patient has been afebrile for some time), a raised CRP and a leucocytosis. Although it is a diagnosis of exclusion, knowledge of its common incidence may prevent over-investigation of patients. Anti-inammatory medication will relieve symptoms, and the course is benign and self-limiting in general. A pericardial effusion has the potential to cause more serious effects secondary to tamponade, but these features would be clinically manifest if signicant. Drug allergy (particularly penicillin) can cause this type of rash, but is often associated with an eosinophilia. It is much less common then IAC. Partially treated meningitis, or recurrent meningococcal disease are unlikely as the patient has had appropriate antibiotic therapy. Patients can have meningococcal disease more than once in their life, as there are different serotypes, but it would

PAEDIATRICS AND CHILD HEALTH 19:1

51

2008 Elsevier Ltd. All rights reserved.

SELF-ASSESSMENT

also prompt a search for defects in complement production or opsonisation, which are associated with recurrent meningococcal and pneumococcal infections. Question 2 (a)  His ventilator settings should be changed Cerebral blood ow is very closely associated with arterial partial pressure of carbon dioxide (pCO2). Increased pCO2 causes cerebral vasodilatation, and the increased cerebral blood ow can cause an increase in intracranial pressure if there is a lack of intracranial space to accommodate it, or a failure of normal cerebral circulatory autoregulation. In ventilated patients who are at risk of raised intracranial pressure, such as those with suspected meningitis, the target pCO2 is 4.55.0 kPa. This patient should have a change to ventilator parameters that reduce his minute ventilation, and his pCO2 re-checked 30 minutes after the change. This presenting feature makes meningitis a possibility as an underlying diagnosis, especially given the appearances of meningeal enhancement. However, this patient was drowsy, and there is a possibility of raised intracranial pressure, which is a contraindication to lumbar puncture. Raised intracranial pressure may be present even if not demonstrated on a CT of the head.2 It would be safer to treat for meningo-encephalitis and perform a lumbar puncture once the childs conscious level is shown to have improved the biochemical and cellular changes within cerebrospinal uid (CSF) will persist for 23 days after antibiotic treatment has started.3 His lactic acid levels are raised, but this is very common after a prolonged tonic clonic seizure, and his normal heart rate, perfusion and mean blood pressure would re-assure you that he does not have shock, and does not require inotropes. Likewise, there is no indication for a uid bolus. A popular misconception is that routine uid restriction should be used in meningitis. This is not supported by published evidence this patient will be started on routine uid requirements, subsequently adjusted on the basis of clinical and laboratory information. (b)  Blood lm The striking feature of this set of blood results is renal failure and an acute drop in haemoglobin and platelet count. The latter two often occur in sepsis secondary to a dilutional effect from uid resuscitation, and disseminated intravascular coagulation. However, his coagulation prole is not supportive of disseminated intravascular coagulation. Haemolytic-uraemic syndrome (HUS) has to be a strong possibility, and a blood lm will demonstrate the red cell fragmentation typical of this microangiopathic haemolytic anaemia. (c)  Haemolytic uraemic syndrome pneumococcal Although E. coli H7:0157 is the most common cause of HUS in children, other bacteria such as Shigella dysenteriae produce the verocytotoxin (or shiga-like toxin) to produce HUS. The toxin acts on vascular endothelial cells to produce a multisystem vasculitis.

Streptococcus pneumoniae produces the enzyme neuraminidase. This cleaves N-acetylneuraminic acid from cell membrane surfaces exposing a cryptantigen, ThomsenFreidenreich antigen (T-antigen), on red blood cells, platelets and glomerular capillaries. Most people possess a naturally occurring antibody to this antigen, and the cells are attacked and destroyed. Some hospital laboratories will perform a conrmatory test demonstrating the presence of T-antigen.4 Pneumococcal HUS almost always occurs in children less than 2 years of age. The clinical course is more severe than non-pneumococcal, and the patients are more likely to require dialysis.5 Question 3 (a)  Toxic shock syndrome Toxic shock syndrome (TSS) is caused by toxin-producing strains of Staphylococcus aureus or Streptococcus pyogenes (Group A streptococcus). Toxic shock syndrome toxin-1 (TSST-1) is best known in S. aureus infections, but other toxins are also produced and believed to function as superantigens, causing a powerful stimulatory effect on T-cell proliferation, and increased production of inammatory cytokines like tumour necrosis factor and interleukin-1 (IL-1).6 Clinical criteria have been dened for the diagnosis. For staphylococcal TSS these are: Negative blood throat and CSF cultures (if taken); blood culture may be positive for S. aureus. Negative serology for measles, leptospirosis or Rocky Mountain spotted fever. Major (all four must be met): Fever more than 38.9 C  Rash diffuse macular erythroderma   Hypotension systolic nlood pressure less than 5th centile for age Desquamation 1 or 2 weeks after the acute illness  Multi-system involvement (three must be met) Gastrointestinal: vomiting or diarrhoea at onset  Muscular: severe myalgia or creatine kinase level greater  than twice upper limit of normal Mucous membranes: vaginal, oropharyngeal, or  conjunctival hyperaemia  Renal: urea or creatinine more than twice the upper limit of normal Hepatic: bilirubin, ALT or AST upper limit of normal  Platelets less than 100 109/l  Altered consciousness when fever and hypotension not  present The denition of staphylococcal TSS does not require the isolation of S. aureus, but that for streptococcal TSS requires isolation of Group A streptococcus from a normally sterile site (e.g. blood, CSF) to make a denite diagnosis, or isolation from a non-sterile site (e.g. throat, skin) to make a diagnosis of a probable case. Clearly, because of the requirement for desquamation, the denitive diagnosis cannot be made while treatment is being initiated, so management is based on clinical suspicion.

PAEDIATRICS AND CHILD HEALTH 19:1

52

2008 Elsevier Ltd. All rights reserved.

SELF-ASSESSMENT

Although this patient had a suspected entry wound for infection, a positive site for primary infection is less common in staphylococcal TSS than it is in streptococcal. This patient grew S. aureus from blood culture and had a desquamating rash shortly before discharge from the PICU where he was admitted. Kawasaki disease is also believed to be a superantigenmediated disease, so clinical features are similar. However, it is most common in children less than 1 year old and very rare over the age of 5 years. Septic arthritis is usually associated with restricted joint movement. Necrotising fasciitis is characterised by severe pain around the infection site, and a rapidly spreading cellulitis the degree of pain in the early stages is often disproportionate to the clinical appearances. StevensJohnson syndrome produces a characteristic rash with target lesions and mucosal ulceration. (b)  Treat with intravenous clindamycin The general principles of caring for critically ill patients apply, suing uid boluses and inotropes are necessary to maintain organ perfusion and oxygenation. Many patients will require to be treated in an intensive care unit. At this stage in treatment, the likely diagnosis is TSS, although whether it is streptococcal or staphylococcal is not certain. Clindamycin is known to suppress toxin production in both types, and so is added in to the antibiotic regimen. Antibiotic treatment is continued for a minimum of 10 days, but may be longer if the source of primary infection requires it (e.g. osteomyelitis). Intravenous immunoglobulin may have a role, but evidence is not compelling. It is currently recommended for cases of TSS that are not responding to conventional treatment.

There is no place for high dose steroids in this condition. Steroids may be used if there is inotrope resistant hypotension, but in physiological replacement doses in the manner recommended for treating septic shock. The issue of whether this patient has a septic arthritis should be resolved, but it is not his most urgent problem, and will not change immediate management. Given the lack of impairment of joint mobility it is very unlikely.

REFErENCES 1  Goedvolk CA, IA von Rosenstiel, Bos AP. Immune complex associated complications in the subacute phase of meningococcal disease: incidence and literature review. Arch Dis Child 2003; 88: 927930. 2 Rennick G, Shann F, de Campo J. Cerebral herniation during bacterial meningitis in children. BMJ 1993; 306: 953955. 3 Riordan FAI, Cant AJ. When to do a lumbar puncture. Arch Dis Child 2002; 87: 2357. 4 Cabrera GR, Fortenberry JD, Warshaw BL, et al. Hemolytic uremic syndrome associated with invasive Streptococcus pneumoniae infection. Pediatrics 1998; 101: 699703. 5  Brandt J, Wong C, Mihm S, et al. Invasive pneumococcal disease and hemolytic uremic syndrome. Pediatrics 2002; 110: 371376. 6 American Academy of Pediatrics. Toxic shock syndrome. In: Pickering LK, ed. Report of the Committee on Infectious Diseases. 27th Edn. Elk Grove Village, IL: Red Book; 2006. American Academy of Pediatrics.

PAEDIATRICS AND CHILD HEALTH 19:1

53

2008 Elsevier Ltd. All rights reserved.

SELF-ASSESSMENT

Self-assessment
Questions
Case 1 A 3-year-old boy presents to A&E with fever of 39C, HR of 140/min, capillary rell time of 34 s and slightly drowsy. On further questioning, it is revealed that he had returned 2 weeks previously from an East African safari, and had taken the recommended malaria prophylaxis. 1.  What are the two most likely diagnoses? Choose TWO answers from the following options: A. Plasmodium falciparum malaria B. Plasmodium vivax malaria C. Bacterial sepsis D. Inuenza E. Dengue fever 2.  Which of the following investigations should have priority? Chose ONE answer from the following options: A. Malarial blood lm B. Blood glucose C. Arterial blood gas D. FBC 3.  What should be the immediate management of this child? Chose ONE answer from the following options: A. Intravenous infusion of quinine sulphate B. Exchange transfusion C.  Fluid restriction to 80% maintenance and intravenous quinine D. Bolus of 20 ml/kg colloid or 0.9% saline. 4.  With what denitive antimalarial treatment should this child be treated? Chose ONE answer from the following options: A. Parenteral quinine for 7 days B. 3-day course of chloroquine C. Parenteral artesunate D.  Intravenous quinine followed by a full course of oral medication with either meoquine, proguanil with atovaquone, or artemeter with lumefantrine Case 2 A 65-year-old woman visiting her family from India is admitted with a febrile illness and cough. She has a caseating lesion in the left upper lobe on chest X-ray and a sputum smear is positive for acid-fast bacilli. She had been staying with her daughter and grandchildren, who are 3 weeks, 18 months and 4 years old, respectively. 1.  The 3-week-old baby is well and thriving. Which management plan should be followed? Chose ONE answer from the following options: A. Give BCG and keep under regular outpatient follow-up B.  Perform a Mantoux test and if positive, treat with full antituberculous treatment with isoniazid, rifampicin and pyrazinamide C.  Start on isoniazid 5 mg/kg and then perform a Mantoux test after 3 months treatment D.  See in regular outpatients until he is 6 months old and then perform a Mantoux test The 18 month old, who had a BCG when she was 1 week 2.  old, is well when rst seen. Which management plan should be followed for her? Chose ONE answer from the following options: A. Keep under regular outpatient follow-up B. Perform a Mantoux test C.  Start on isoniazid 5 mg/kg and then perform a Mantoux test after 3 months treatment The Mantoux test is positive at 17 mm. The child is well with height and weight on the 75th centile and she has no fevers or night sweats. A chest X-ray shows prominent right hilum and partial collapse consolidation of the right upper lobe. 3.  What course of management should be followed? Chose ONE answer from the following options: A.  The child is well and should be treated for latent TB with isoniazid and rifampicin for 3 months B.  The X-ray changes are probably due to intercurrent infection and should be treated with a course of oral co-amoxyclav with a follow-up X-ray C.  The chest X-ray shows evidence of clinical disease and the child should be treated with 6 months antituberculous treatment 4.  The 4-year-old had a BCG as a baby and has a visible scar on his upper arm. He is well. Which management plan should be followed for him? Chose ONE answer from the following options: A. Perform a Mantoux test B. No further action required as he has had a BCG C.  The child is well and should be treated for latent TB with isoniazid and rifampicin for 3 months The child is reviewed with the results of the Mantoux test, which is 16 mm. He goes on to have a gamma-interferon test which is negative. 5.  What should happen now? Chose ONE answer from the following options: A. Perform a chest X-ray and treat for active TB

Jennifer Evans MD MRCPCH is a Consultant Paediatrician, University Hospital of Wales, Cardiff, Wales, UK.

PAEDIATRICS AND CHILD HEALTH 19:2

93

2008 Published by Elsevier Ltd.

SELF-ASSESSMENT

B.  Inform and advise his parents, and discharge him from the clinic C. Treat for latent TB and X-ray again in 6 months D. Perform a further BCG Case 3 A 3-year-old boy is brought to A&E. He is with his grandmother who reports that he fell off his bike earlier in the day, but she did not think it was serious. He is now lethargic and complaining of left leg pain. On examination he is febrile at 38C, HR of 160/min and is complaining of leg pain, but there is a full range of movement. He has two small bruises on his legs and two non-blanching petechiae on his abdomen. 1.  What is the immediate management? Chose ONE answer from the following options: A. Prescribe paracetamol for the pain and discharge home B.  Place his leg in a cast, give paracetamol and await a trauma opinion in the morning C. A  ssess his airway patency and breathing, consider giving high ow oxygen, further assess for signs of shock, obtain IV access and give a bolus of 20 ml/kg uid D. T  ake a blood culture and start IV antibiotics with ucloxacillin The rash extends and several purpura appear on his legs. The child however is much more alert and is talking normally to his grandmother. His HR remains 160170/min and capillary rell time centrally is 4 s. 2.  What is the optimal further management? Chose ONE answer from the following options: A. U  rgent MRI of his leg to exclude osteomyelitis B.  A diagnosis of meningococcal disease is likely so he should be given IV penicillin, corticosteroids and a lumbar puncture C. P  lace on IV antibiotics and uids and assess 2 hourly for signs of shock D. G  ive a further bolus of 20 ml/kg of colloid, IV ceftriaxone and make contact with a PICU and anaesthetics 3.  Which two of the following blood tests will be most useful: A. Arterial blood gas B. Clotting screen C. Liver function tests D. CRP E. CSF culture 4.  He has two brothers, aged 5 and 7, and his mother is pregnant. How should they be managed? Chose ONE answer from the following options: A.  Children should receive rifampicin 10 mg/kg and the mother rifampicin 600 mg Children should receive rifampicin 10 mg/kg and the B.  mother 500 mg ciprooxacin as she is pregnant

Children should receive rifampicin 10 mg/kg and the C.  mother ceftriaxone 250 mg in a single IM dose

Answers
Case 1 1. A and C The most likely diagnoses are Plasmodium falciparum malaria or bacterial sepsis, which clinically can appear to be very similar, both presenting with shock. The presence of shock makes inuenza less likely. That the boy has taken malaria chemoprophylaxis is no reassurance, as this cannot prevent all cases of malaria. Plasmodium vivax is prevalent on the Indian subcontinent and in Central America but is only rarely seen in Africa as it preferentially invades erythrocytes bearing the Duffy blood group antigen, rarely found in the African population. Worldwide Plasmodium falciparum is responsible for cases of severe and complicated malaria, which untreated has a signicant mortality. Dengue fever is also associated with fever and shock but is much more common in the tropical areas of Asia and America and is unusual in East Africa. 2. B Although all four of these investigations are necessary, a blood glucose is urgent in severe malaria where hypoglycaemia can occur, causing reduced conscious level and convulsions. 3. D This child has several features indicating he is at high risk and in need of urgent supportive management. These include a depressed conscious level and evidence of shock with tachycardia and a prolonged capillary rell time. The emergency assessment and management of the child should follow the structured approach advocated in the APLS guidelines. Emergency management must not be delayed while the diagnosis of malaria is conrmed. The initial management of shock should include a bolus of 20 ml/kg of colloid or normal saline. There is no evidence that exchange transfusion has a role in the initial management of children with suspected malaria and it may distract from simpler resuscitation measures. 4. D Parenteral quinine remains the antimalarial treatment of choice for patients with severe falciparum malaria and should be prescribed for 7 days. However, children often recover clinically before then and the course may be shortened by switching to a full oral course of an appropriate non-quinine medication. Oral quinine has a bitter taste and is associated with poor compliance; therefore, other oral medications are

PAEDIATRICS AND CHILD HEALTH 19:2

94

2008 Published by Elsevier Ltd.

SELF-ASSESSMENT

recommended. Parenteral artesunate has been shown to be superior to quinine in the treatment of severe malaria in adults in South-East Asia. These results cannot however be extrapolated to children and a multicentre study in children is currently ongoing. Chloroquine resistance is now widespread and is no longer the rst-line treatment for falciparum malaria.

FURTHER REaDING 1 National Collaborating Centre for Chronic Conditions. Tuberculosis: Clinical diagnosis and management of tuberculosis, and measures for its prevention and control. London: Royal College of Physicians, 2006. 2 Taylor RE, Cant AJ, Clark JE. Potential effect of NICE tuberculosis guidelines on paediatric tuberculosis screening. Arch Dis Child 2008; 93: 200203.

FURTHER REaDING Maitland K, Nadel S, Pollard AJ, Williams TN, Newton CRJC, Levin M. Management of severe malaria in children: proposed guidelines for the United Kingdom. BMJ 2005; 331: 337343.

Case 3 1. C 2. D 3. A and B 4. C Meningococcal disease remains an important cause of mortality in children in the UK. Studies have shown that an increased risk of death is associated with the failure to recognize complications, such as shock or raised intracranial pressure, and how ill children are; being managed by unsupervised junior doctors and by non-paediatric trained staff; and management that is often not sufciently aggressive, as indicated by a failure to use enough inotropes in septicaemic patients.1 A key feature in this patient is the severe limb pain in the absence of any other physical signs in the limb, which is a well-established phenomenon in meningococcal disease and must not be attributed to a possible accident, especially in the presence of other clues such as fever. The pain can be very severe and children have been mistakenly put into plaster to treat presumed fractures. Underlying disease may be very advanced by the time a rash appears. The rapidly evolving haemorrhagic rash may be a very late sign. The appearance of purpura in this boy (haemorrhagic lesions of >2 mm) is a further sign of advancing disease. Shock is a clinical diagnosis and is clearly present in this patient. The signs are a result of circulatory failure but, as here, in early shock the child may still be alert and have a normal blood pressure. The early signs of shock include tachycardia and a prolonged capillary rell time. If the clinical response to a bolus of 20 ml/kg of uid is short-lived or absent, and shock does not improve or progresses, large volumes may be required (over 60 ml/kg in the rst hour). In this case, there is a signicant risk of pulmonary oedema, so elective tracheal intubation and mechanical ventilation should be initiated, even if there are no signs of respiratory failure, to optimize oxygenation, reduce the work of breathing and improve cardiac function. Therefore, it is important to alert senior staff early. The presence of a metabolic acidosis with a base decit of less than 5 will give a guide to the severity of illness and the need for further uid and bicarbonate. Coagulopathy is also a marker of severity of disease and if deranged indicates the need for fresh frozen plasma. Hypoglycaemia (< 3.3 mmol/L) is common and should be corrected.2,3

Case 2 1. C 2. B 3. C 4. A 5. B The management of close contacts of individuals with active tuberculosis have to be contacted and screened and are managed according to the NICE guidelines.1 The management is determined by the age of the child and whether they have received a BCG. The 3 week old baby: Neonates who have been in close contact with people with sputum smear-positive TB who have not received at least two weeks anti-tuberculosis drug treatment should be treated as follows: The baby should be started on isoniazid 5 mg/kg and then a Mantoux test performed after three months treatment. If the Mantoux test is positive (6 mm or greater) the baby should be assessed for active TB with clinical assessment and chest X ray. If this assessment is negative, then isoniazid should be continued for a total of six months. If the test is negative (less than 6 mm), then isoniazid should be stopped and a BCG vaccination performed. The 18 month old child: BCG-vaccinated children aged older than four weeks but younger than two years, in close contact with people with sputum smear-positive respiratory TB, should have a Mantoux test. This is considered to be positive if measures 15 mm or more and the child should then be assessed for active TB. An abnormal chest X-ray in an asymptomatic child is a sign of active disease requiring six months treatment with at least three drugs for the initial two months. The seven year old child had had prior BCG but the mantoux measured more than 15 mm so is considered to be positive. The use of the interferon-gamma blood test helps to distinguish between mantous reactivity due to prior BCG or to actual infection with Mycobacterium tuberculosis.2 A negative test in this child indicated the mantoux test was probably due to the prior BCG and not to either active or latent tuberculosis.

PAEDIATRICS AND CHILD HEALTH 19:2

95

2008 Published by Elsevier Ltd.

SELF-ASSESSMENT

All cases of meningococcal disease should be reported to public health and close contacts given prophylaxis to prevent secondary cases.

FURTHER REaDING 1 Ninis N, Phillips C, Bailey L, et al. The role of healthcare delivery in the outcome of meningococcal disease in children: case

control study of fatal and non-fatal cases. BMJ 2005; 330: 1475. 2  Pollard AJ, Nadel S, Ninis N, Faust SN, Levin M. Emergency management of meningococcal disease: eight years on. Arch Dis Child 2007; 92: 283286. 3 Ninis N, Nadel S, Glennie L. Clinicians Guide to Recognition and Early Management of Meningococcal Disease in Children. An e learning tool accessible from the Meningitis Research Foundations website www.meningitis.org.

PAEDIATRICS AND CHILD HEALTH 19:2

96

2008 Published by Elsevier Ltd.

SELF-ASSESSMENT

Self-assessment
Case 1 A previously well, 11-year-old boy presented to the emergency department with a 4-day history of being unwell. His symptoms started with a headache and redness of the left eye. After 1 day he developed a fever of 40 C together with an erythematous skin rash, a sore throat and generalized abdominal pain. He was seen by his GP 3 days prior to admission, was diagnosed with tonsillitis and sinusitis, and was commenced on oral penicillin. He came to the emergency department today as his temperature was increasing, his rash had become more orid, and he had developed painful swallowing. He was drinking less, passing minimal amounts of urine and had three loose stools. His initial observations are as follows: weight 48 kg, temperature 37.7 C, heart rate 130 beats/min, blood pressure 87/33 mmHg, respiratory rate 40 breaths/min, capillary rell time less than 2 s, saturations 99% in air. On examination he looks unwell, with bilateral red itchy eyes and red lips, and his tongue is very dry and cracked. He has bilateral tender cervical lymphadenopathy. His rash is erythematous, warm and blanching. He has bilateral enlarged tonsils with pus in the pharynx (Figure 1). Initial investigations showed: Haemoglobin White blood cell count Neutrophil count Platelet count Sodium Potassium Urea Creatinine C-reactive protein 11.8 g/dl (9.614.8) 6.6 109/L (5.014.0) 5.79 109/L (1.58.0) 116 109/L (200420) 125 mmol/L (135145) 3.2 mmol/L (3.55.0) 17.0 mmol/L (2.97.5) 113 mmol/L (44108) 325 mg/L (05)

What is the most likely diagnosis? Choose ONE answer 1.  ONLY from the following: A.  Kawasakis disease B.  Tonsillitis and penicillin allergy C.  Toxic shock syndrome D.  Measles E.  Scarlet fever 2.  What is the most important initial management? Choose ONE answer ONLY from the following: A.  Intravenous immunoglobulin (IVIG) B.  Corticosteroids C.  Chlorpheniramine D.  Intravenous benzylpenicillin E.  Intravenous 0.9% saline uid boluses His condition remains resistant to the intervention above and he requires intubation and commencement of mechanical ventilation and transfer to the paediatric intensive care unit. His current observations include a heart rate of 150 beats/min and a mean blood pressure of 50 mmHg. 3.  Which of the following management options should be used next? Choose ONE answer ONLY from the following: A.  Furosemide B.  Aspirin Inotropic support C.  D.  Corticosteroids E.  Intravenous immunoglobulin (IVIG)

Figure 1 Rash found on examination.

Mehrengise Cooper FRCPCH is a Consultant Paediatric Intensivist, Paediatric Intensive Care Unit, St Marys Hospital, Paddington, London W2 1NY, UK. Shermina Sayani MBBS BSc MRCPCH is a Fellow in Medical Education, Imperial College Healthcare NHS Trust, Praed Street, London W2 1NY, UK.

Case 2 A previously t and well, 12-year-old girl is admitted to the paediatric ward. She has been unwell for 1 day with vomiting and abdominal pain. In the emergency department, she was seen by the SHO who referred her to the surgeons. She is assessed on the paediatric ward and is dehydrated with dry mucous membranes and sunken eyes, and has generalized abdominal tenderness. The surgical team has suggested ordering an abdominal ultrasound. Unfortunately, it was not possible to obtain intravenous access in the Emergency department, but this is undertaken successfully on the ward.

PAEDIATRICS AND CHILD HEALTH 19:3

145

2009 Elsevier Ltd. All rights reserved.

SELF-ASSESSMENT

Her initial observations are as follows: temperature 37.9C, heart rate 120 beats/min with normal pulses, blood pressure 90/50 mmHg, respiratory rate 40 breaths/min, capillary rell time 34 s, saturations 95% in room air. A venous blood gas taken when intravenous access was gained shows: pH pCO2 pO2 HCO3 Base excess Sodium Potassium Glucose 7.18 (7.357.45) 2.5 kPa (4.76.0) 5.3 kPa (8.010.0) 9 mmol/L (22.026.0) 18 (2.0 to + 2.0) 134 mmol/L (135145) 4.5 mmol/L (3.55.0) ***

Yesterday he only took three of his normal feeds. He was born at term with no complications. On examination his heart rate is 140 beats/min, with a respiratory rate of 70 breaths/min and saturations of 92% in room air. He has moderate intercostal recession and, on auscultation, crepitations and wheeze throughout both lung elds. His temperature is 37.5 C. He is pink, active and interacting with his mother. 1.  What is the most appropriate next step? Choose ONE answer ONLY from the following: A.  Capillary blood gas B.  Chest X-ray Blood culture C.  D.  Admit and start supportive treatment E.  Salbutamol nebulizer You are called to review him later that evening as he has deteriorated, becoming more tachypnoeic with an increasing oxygen requirement. You take a capillary blood gas and organise a chest X-ray (Figure 2). A nasopharyngeal aspirate (NPA) is positive for respiratory syncytial virus (RSV). His venous blood gas shows: pH PCO2 Bicarbonate Base excess 7.30 (7.357.45) 6.8 kPa (4.76.0) 25 mmol/L (22.026.0) 3 (2.0 to + 2.0)

Which of the following investigations will help to conrm the 1.  diagnosis? Choose ONE answer ONLY from the following: A.  Ultrasound abdomen B.  Full blood count C.  Urine microscopy and culture D.  Stool culture E.  Urine dipstick 2.  What is the most appropriate initial uid prescription? Choose ONE answer ONLY from the following: A.  IV 0.45% saline and 5% dextrose maintenance + decit over 24 h B.  IV 0.45% saline and 5% dextrose with potassium as maintenance over 24 h C.  Trial of NG Dioralyte D.  10 ml/kg normal saline bolus, followed by normal saline with potassium as maintenance + 10% decit over 48 h E.  10 ml/kg normal saline bolus followed by 0.45% saline and 5% dextrose maintenance + decit over 48 h Note decit = % dehydration body weight (kg) Later that night you review the patient as she has been complaining of a headache. She has been given some paracetamol and by the time you arrive she has become drowsy. Her observations show the following: heart rate 70 beats/min, blood pressure 150/80 mmHg, capillary rell time less than 2 s and BM 10 mmol/L. You assess her Glasgow coma score (GCS) and it is 12/15. 3.  What should you do next? Choose ONE answer ONLY from the following: A.  Return to review her again in a couple of hours B.  A full neurological exam in the morning when she is more awake C.  CT head scan D.  Repeat blood gas Treat raised intracranial pressure with mannitol or 3% E.  saline Case 3 A 9-month-old boy is brought to the Emergency department by his mother. He has a 2-day history of poor feeding and coryza.

Figure 2 Chest X-ray of patient.

2.  Which of the following is the appropriate next step? Choose ONE answer ONLY from the following: A.  Atrovent nebulizer B.  Corticosteroids C.  Adrenaline nebulizer

PAEDIATRICS AND CHILD HEALTH 19:3

146

2009 Elsevier Ltd. All rights reserved.

SELF-ASSESSMENT

D.  Stop feeds and commence intravenous uids E.  Salbutamol nebulizer His nurse tells you that over the previous 3 h he has had three episodes of desaturation to 70% for 30 s, and required stimulation and increased oxygen to return to more than 95% in nasal cannulae oxygen. 3.  Which other therapeutic option will be benecial to this patient? Choose ONE answer ONLY from the following: A.  Intravenous antibiotics B.  Start nasal CPAP Diuretics C.  D.  Review in 1 h with repeat capillary blood gas E.  Arrange for intubation and ventilation Case 4 A 7-year-old boy presents to the Emergency department with a 10-day history of fever, vomiting, mainly in the mornings, a headache which is not improving with regular ibuprofen, and he is more sleepy than normal. He has had no history of loss of consciousness. He has been feeling weak, has not been walking for 1 day and has started behaving strangely. His initial observations are: temperature 36 C, pulse 92 beats/min, mean blood pressure 80 mmHg, respiratory rate 30/min, saturations 99% in air, capillary rell time less than 2 s. His BM is 6.2 mmol/L and his urine dipstick is NAD. He is alert, but moaning in pain with a GCS of 15/15. He has neck stiffness with a negative Kernigs sign. His cranial nerve examination is normal. Of note, on examining his reexes, he has downgoing plantars. He has cervical lymphadenopathy and his tonsils are enlarged with an exudate present. His initial venous blood gas is: pH pCO2 HCO3 BE 7.52 (7.357.45) 3.7 kPa (4.76.0) 22.9 mmol/L (22.026.0) 1.1 (2.0 to + 2.0)

C.  Discharge home with penicillin V for the tonsillitis Ask to return in 2 days if there is no improvement D.  E.  Admit to the ward for observation While you are writing in your notes, before you have carried out your management plan, the nurse calls you over to review him. His vital signs are the following: temperature 37 C, heart rate 68 beats/min, mean blood pressure 110 mmHg, respiratory rate 3040 breaths/min with an irregular pattern, capillary rell time less than 2 s and saturations 92% in air. His GCS is 9. 2.  What would you do next? Choose ONE answer ONLY from the following: A.  CT head scan and lumbar puncture B.  Give intravenous mannitol at a dose of 0.250.5 g/kg Intubate and commence mechanical ventilation C.  D.  Commence neurological observations every 15 min and review after 1 h E.  Fluid restriction After an initial improvement, he has a further deterioration, is now making sterterous sounds and has developed focal seizures affecting his left leg. He is intubated and commenced on mechanical ventilation. His bloods are repeated and his serum sodium is 128 mmol/L. 3.  What therapeutic manoeuvre will you institute next? Choose ONE answer ONLY from the following: A.  Give intravenous mannitol at a dose of 0.250.5 g/kg B.  Fluid restriction to 60% maintenance Dose of intravenous 3% saline at a dose of 3 ml/kg C.  D.  Dose of intravenous lorazepam at 100 g/kg E.  Referral to neurosurgeon for placement of ICP bolt Case 5 A 7-day-old baby presents to the emergency department with a 24-h history of poor feeding, and episodes of cyanosis. She was born at full-term by spontaneous vaginal delivery following an uncomplicated pregnancy with a birth weight of 3.2 kg. Her parents are non-consanguinous and she has a 2- year-old sibling who is well. She was discharged home on the second day of life, and has been breast feeding well. On examination she is pale, and lethargic. Her weight is 3.5 kg, and she is tachypnoeic with a respiratory rate of 80 breaths/min. Her heart rate is 180 beats/min and her capillary rell time is 5 s. She has a liver palpable to 4 cm below the costal angle. It has been difcult to feel her femoral pulses and obtain a blood pressure in her lower limbs. Intravenous access is gained and an initial venous blood gas shows: pH pCO2 pO2 HCO3 Base excess Lactate 7.07 (7.357.45) 3.7 kPa (4.76.0) 2.8 kPa 10 mmol/L (22.026.0) 20 (2.0 to + 2.0) 12 mmol/L (02.0)

Initial investigations showed the following: Hb White blood cell count Neutrophils Platelet count Sodium Potassium Urea Creatinine C-reactive protein 12.2 g/dl (9.614.8) 23.7 109/L (5.014.0) 16 109/L (1.58.0) 555 109 g/dl (200420) 130 mmol/L (135145) 3.4 mmol/L (3.55.0) 2.2 mmol/L (2.97.5) 61 mmol/L (44108) 6.5 mg/L (05.0)

What is your management plan at this point? Choose ONE 1.  answer ONLY from the following: A.  Explain this is probably a viral infection, reassure and discharge home B.  CT head scan and lumbar puncture

PAEDIATRICS AND CHILD HEALTH 19:3

147

2009 Elsevier Ltd. All rights reserved.

SELF-ASSESSMENT

1.  Which of the following best describes this blood gas? Choose ONE answer ONLY from the following: A.  Respiratory acidosis B.  Respiratory alkalosis C.  Metabolic alkalosis with respiratory compensation D.  Metabolic acidosis with respiratory compensation E.  Respiratory acidosis with metabolic compensation 2.  What is the likely diagnosis? Choose ONE answer ONLY from the following: A.  Bronchiolitis B.  Neonatal sepsis C.  Transposition of the great arteries D.  Coarctation of the aorta E.  Metabolic disorder The baby deteriorates developing marked increased work of breathing followed by a prolonged apnoea and is fully resuscitated, with intravenous uids and is intubated and commenced on mechanical ventilation. 3.  What therapy would be most valuable next? Choose ONE answer ONLY from the following: A.  Infusion of prostaglandin B.  Inhaled nitric oxide Intravenous antibiotics C.  D.  Inotropic support E.  Balloon atrial septostomy

among organs. He is at risk of developing decompensated shock and requires urgent uid resuscitation. The rstline uid should be 0.9% saline and should be given as a uid bolus of 20 ml/kg, maintaining close observations throughout. 3. C This child requires inotropic support; the low diastolic blood pressure indicates peripheral vasodilation and an inotrope must be commenced. IVIG (intravenous immunoglobulin) may be used for TSS and there is some evidence to suggest immunoglobulins directed against the toxins are an effective additional therapy, and useful when conventional therapies do not control the symptoms. It is thought that IVIG probably provides the antibodies to neutralize the antitoxin. The use of corticosteroids in shock is controversial and should be discussed with a paediatric intensive care unit. As a therapy for TSS, they are not effective.
FURTHER REaDING Barry W, Hudgins L, Donta ST, Pesanti EL. Intravenous immunoglobulin therapy for toxic shock syndrome. JAMA 1992;267:33153316. Buchdahl R, Levin M, Wilkins B, et al. Toxic shock syndrome. Arch Dis Child 1985;60:563567. Dellinger RP, Levy MM, Carlet JM, et al. Surviving Sepsis Campaign: international guidelines for the management of severe sepsis and septic shock 2008. Crit Care Med 2008;36:297327. Williams GR. The toxic shock syndrome. BMJ 1990;300:960.

Answers
Case 1 1. C Toxic shock syndrome (TSS) is an acute, toxin-mediated febrile illness that rapidly leads to multiorgan failure, with serious morbidity and mortality. This patient meets the clinical criteria for diagnosis of TSS, i.e. a fever of at least 38.9 C, a diffuse macular erythematous rash and hypotension. For diagnostic completion, toxin action on at least three systems must be demonstrated with either diarrhoea or vomiting, myalgia or raised creatinine kinase, mucous membrane hyperaemia, elevated concentrations of blood creatinine or urea, elevated transaminases, thrombocytopenia and confusion or drowsiness. TSS is classically associated with the use of tampons, but it is well described in other circumstances. TSS is caused by Staphylococcus aureus and Group A -haemolytic streptococcus. The bacterial toxins are known as superantigens due to their ability to bypass the usual steps seen in the antigen-mediated immune response and directly to activate the immune system. 2. E This child is shocked, which is apparent from his tachycardia, tachypnoea, hypotension (note the low diastolic pressure) and low urine output. Typically this is described as distributive shock and occurs when blood is redistributed

Case 2 1. E This child is dehydrated with abdominal pain and vomiting. In addition, she is tachypnoeic, has a metabolic acidosis and her glucose has not been recorded. Urine dipstick would conrm glycosuria and ketonuria, giving a diagnosis of diabetic ketoacidosis (DKA). A laboratory glucose will conrm this but it was not possible in the case described. DKA can sometimes present with features consistent with an acute abdomen. It is therefore important to dipstick the urine of any child who has abdominal pain or is vomiting. 2. D She is 10% dehydrated, vomiting and acidotic, and requires IV uids. In the management of DKA an initial uid bolus may be given slowly if required followed by maintenance uids. In order to correct the percentage of dehydration, the decit of 10% is added to the uid requirement and this should run over 48 h so as to not lead to any large uid shifts and electrolyte changes. Maintenance uids are always given initially as normal saline. Potassium may be required after initial resuscitation as insulin drives potassium uptake into cells; thus, serum electrolytes must be monitored regularly.

PAEDIATRICS AND CHILD HEALTH 19:3

148

2009 Elsevier Ltd. All rights reserved.

SELF-ASSESSMENT

An intravenous insulin infusion should be commenced at 0.050.1 U/kg/h. The infusion should be titrated to aim for a fall of glucose no more than 5 mmol/L/h. This child must have hourly observations of her vital signs, including neurological observations. She should have hourly to hourly blood glucose measurements and 4 hourly electrolytes. Regular monitoring and titration of insulin and uids is essential in order to prevent deterioration. 3. E This child has developed some signs of raised intracranial pressure, most likely due to cerebral oedema. This is evident with her headache, reduced GCS, increasing blood pressure and dropping heart rate. This must be treated as soon as possible with a hyperosmolar agent. Her blood glucose has dropped to 10 mmol/L. This rapid reduction in blood glucose and any accompanying uid shifts may be associated with the development of intracranial hypertension in DKA.

it may be useful to administer these agents at least on a trial basis. 3. B The use of CPAP is appropriate where there is a respiratory deterioration and apnoeas. If there is no benet from CPAP, then intubation and mechanical ventilation are indicated.

FURTHER REaDING Meates-Dennis M. Bronchiolitis. Arch Dis Child Educ Pract Ed 2005;90:ep8186.

FURTHER REaDING Dunger DB, Sperling MA, Acerini CL, et al. ESPE/LWPES consensus statement on diabetic ketoacidosis in children and adolescents. Arch Dis Child 2004;89:188194. Glaser N, Barnett P, McCaslin I, et al. Risk factors for cerebral edema in children with diabetic ketoacidosis. N Engl J Med 2001;344:264269. Wolfsdorf J, Craig ME, Daneman D, et al: ISPAD Clinical Practice Consensus Guidelines 20062007. Diabetic ketoacidosis. Pediatr Diabetes 2007;8:2843.

Case 4 1. B This child has signs and symptoms consistent with a meningoencephalitis. Initially the child is stable and at this point it is advisable to perform a CT head to rule out a space-occupying lesion, e.g. a cerebral abscess, and if appropriate to proceed to perform a lumbar puncture. A normal CT brain scan does not rule out raised intracranial pressure. This is safe to do as his neurological examination is normal and there are no other contraindications. 2. B He has developed signs of raised intracranial pressure with a decreased level of consciousness, raised blood pressure and bradycardia. At this point the lumbar puncture is not indicated because the risk of cerebral herniation is high. Hyperosmolar therapy is used for treating raised intracranial pressure in the acute setting. The agents used for hyperosmolar therapy are mannitol and 3% saline. Mannitol acts by reducing blood viscosity and therefore blood vessel diameter where there is intact cerebral autoregulation. Mannitol also acts by its osmotic effect. Although no studies on mannitol have been carried out in children, it is used extensively. Hypertonic saline also acts to increase the hypertonicity of cells, and is becoming incorporated into the acute management of intracranial hypertension. 3. D He has developed focal seizures that must be controlled as soon as possible. The algorithm for the management of seizures must be followed. The seizures may be due to intracranial hypertension and treatment can be instituted for this.

Case 3 1. D Bronchiolitis is the most common respiratory illness affecting children under the age of 2 years, with an incidence peaking in the rst year of life. Its typical features start with symptoms of an upper airway viral infection, and over the following 46 days, the lower respiratory tract becomes affected with cough, tachypnoea, hyperination, widespread crackles and wheeze. In November 2006 the Scottish Intercollegiate Guidelines Network (SIGN) published an evidence-based guideline on the management of bronchiolitis. For the hospital management of bronchiolitis, it recommends that all infants with oxygen saturation 92% require inpatient care and that infants with oxygen saturations more than 94% in room air may be considered for discharge. Blood culture and chest X-ray are not necessary. Blood gas measurement is only required in severe cases. 2. D As this child has become more distressed, it is most appropriate to stop gastric feeds and commence intravenous uids. The use of nebulized agents has no scientically dened recommendation; however, in the clinical setting,

FURTHER REaDING Riordan FAI, Cant AJ. When to do a lumbar puncture. Arch Dis Child 2002;87:235237. The Paediatric Accident and Emergency Research Group. RCPCH Guideline November 2005. The Management of a Child with a Decreased Conscious Level. London: Royal College of Paediatrics and Child Health, 2005.

PAEDIATRICS AND CHILD HEALTH 19:3

149

2009 Elsevier Ltd. All rights reserved.

SELF-ASSESSMENT

Case 5 1. D This baby has developed a metabolic acidosis with respiratory compensation. 2. D 3. A Coarctation of the aorta presenting in the neonatal period usually has an acute onset of obstruction to systemic blood ow, leading to left ventricular failure and cardiovascular collapse. A patent ductus arteriosus (PDA) allows blood to pass from the right ventricle to the descending aorta, and when the PDA closes, this leads to acute cardiovascular collapse. This can lead to a baby presenting with severe shock, metabolic

acidosis and end-organ ischaemia. Other left-sided obstructive lesions, including hypoplastic left heart syndrome, may present in a similar way. The baby must be resuscitated on presentation, and for any duct-dependent cardiac lesion this includes an infusion of Prostaglandin in order to open up the duct to allow blood to ow into the descending aorta. These babies must be reviewed by a paediatric cardiologist as soon as possible, in order to perform an echocardiogram to conrm and review the anatomical diagnosis.

FURTHER REaDING Chang AC, Hanley FL, Wernovsky G, Wessel D (eds). Pediatric Cardiac Intensive Care 1998.

PAEDIATRICS AND CHILD HEALTH 19:3

150

2009 Elsevier Ltd. All rights reserved.

SELF-ASSESSMENT

Self-assessment
Questions
Case 1 A 5-day-old term infant presents with a history of multiple apnoeic episodes during which she was noted by her parents to have blue lips and appeared not to be breathing. On examination the infant was alert, afebrile and not dysmorphic; heart sounds were normal at 110/min. The chest was clear with good air entry and the fontanelle was at and soft with a slight pulsation noted; neurological and abdominal examination was normal. The infant continued to have frequent desaturations and a full septic screen was performed but no organism was found. Chest X-ray, head ultrasound, echocardiogram and pH studies were normal. A capillary blood gas showed an elevated pCO2 of 7.58.2 kPa. A sleep study was performed and a section of this is shown below. Each epoch represents 15 s. Multiple similar episodes were seen throughout the study. 2.  Which of the following investigations would be most likely to conrm the diagnosis? Choose ONE answer ONLY from the following: A.  Genetic studies B.  Electroencephalogram C.  Flexible bronchoscopy D.  24-h ECG E.  Echocardiogram 3.  Which of the following would be the most appropriate management option for this infant? Choose ONE answer ONLY from the following: A.  Corticosteroid therapy B.  Invasive positive pressure ventilation C.  Non-invasive positive pressure ventilation D.  Surgical intervention E.  Home oxygen therapy

Nasal air flow

Thorax movement

Abdominal movement Oxygen saturations

Case 2 A 3-year-old boy presents with an acute history of breathlessness and right-sided chest pain. On examination he was found to have signs consistent with a pneumothorax and a chest drain was inserted. In addition he had a long history of a productive cough and recurrent infected eczema. He had received six courses of oral antibiotics in the last year, which helped the cough for a short period before it recurred. He went on to have the following investigations performed. Full blood count Haemoglobin White cell count Neutrophils Lymphocytes Eosinophils Platelets

96 96 96 96 96 96 96 96 96 95 94 92

91 91 91

93

Age

1.  Which of the following is the most likely diagnosis? Choose ONE answer ONLY from the following: A.  Obstructive sleep apnoea B.  Congenital hydrocephalus C.  Infantile spasms D.  Congenital central hypoventilation syndrome E.  Benign periodic breathing

11.1 g/dl 9.3 109/L 40% 36% 15% 786 109/L

Immunoglobulins IgG IgM IgA IgE

6.27 g/L 0.55 g/L 1.02 g/L 10,350 IU/ml

(4.212.4) (0.351.55) (0.251.85) (<120)

Philip Davies MD MA BChir MRCPCH is a Consultant in Paediatric Respiratory Medicine at the Royal Hospital for Sick Children, Yorkhill, Glasgow, Scotland, UK. Sarah Brown MB ChB MRCPCH is a Paediatric Specialist Registrar at the Royal Hospital for Sick Children, Yorkhill, Glasgow, Scotland, UK.

Sweat test Chloride Weight of sweat

40 mmol/L 131 mg

Nitroblue tetrazolium (NBT) test was negative. Below is a thoracic CT scan of the chest taken following pneumothorax and chest drain insertion.

PAEDIATRICS AND CHILD HEALTH 19:4

185

2008 Elsevier Ltd. All rights reserved.

SELF-ASSESSMENT

Chloride Bicarbonate Urea Creatinine CRP Full blood count Haemoglobin White cell count Neutrophils Lymphocytes Platelets

101 mmol/L 28 mmol/L 3.0 mmol/L 36 mol/L 283 mg/L

(95110) (1826) (2.56.0) (1840) (<7)

12.8 g/dl 24.3 109/L 92% 6% 247 109/L

1.  In addition to the pneumothorax and chest drain, what anomaly is shown on the CT scan? Choose ONE answer ONLY from the following: A.  Pneumatocoele B.  Bronchiectasis C.  Pulmonary lobar emphysema D.  Congenital cystic adenomatous malformation E.  Pulmonary sequestration 2.  Which of the following organisms is most likely to be responsible for the lung pathology seen? Choose ONE answer ONLY from the following: A.  Mycobacterium tuberculosis B.  Aspergillus fumigatus C.  Esherichia coli D.  Pseudomonas aeruginosa E.  Staphylococcus aureus 3.  Which of the following options is the most likely underlying diagnosis? Choose ONE answer ONLY from the following: A.  Cystic brosis B.  Job syndrome C.  WiskottAldrich syndrome D.  Primary ciliary dyskinesia Chronic granulomatous disease E.  Case 3 A 4-year-old boy presented to his local hospital with a 5-day history of cough, abdominal pain, pyrexia, malaise and poor oral intake. He was noted to have a respiratory rate of 35 breaths/min with intercostal recession, oxygen saturations of 91% in air and decreased air entry in the left lower region. A chest X-ray was performed which showed opacication of the left lower lobe. The patient was admitted for intravenous antibiotics and oxygen therapy. Blood tests taken on admission gave the following results: Electrolytes Na+ K+

Over the next 4 days, the patients symptoms persisted and, on examination, the chest became stony dull to percussion. Repeat chest X-ray showed a white out of the left side, with apparent scoliosis. A chest ultrasound demonstrated a loculated, parapneumonic effusion. The child was referred to the paediatric tertiary referral centre for further management. 1.  Which of the following is the most likely cause of his hyponatraemia? Choose ONE answer ONLY from the following: A.  Low sodium intake B.  Increased renal sodium excretion C.  Hyponatraemic dehydration D.  Increased sodium dilution E.  High sweat sodium concentrations 2.  Which of the following organisms is most likely to be responsible for his symptoms? Choose ONE answer ONLY from the following: A.  Staphylococcus aureus B.  Streptococcus pneumoniae C.  Klebsiella pneumoniae D.  Mycobacterium tuberculosis E.  Mycoplasma pneumoniae 3.  On arrival at the paediatric tertiary centre, which of the following actions is the most appropriate next step in management? Choose ONE answer ONLY from the following: A.  Surgical decortication B.  Conservative management and observation for 48 h C.  Small bore chest drain insertion with intrapleural brinolytics D.  Large bore chest drain insertion E.  Chest drain insertion (small or large bore) with intrapleural antibiotics Case 4 A 15-year-old boy presents with a recurrent dry cough and breathlessness with activity which has become more obvious over the previous 6 months. Of note in his past medical history, at age 6 he was diagnosed with stage IV neuroblastoma and completed treatment consisting of chemotherapy,

126 mmol/L 3.5 mmol/L

(135145) (3.55.6)

PAEDIATRICS AND CHILD HEALTH 19:4

186

2008 Elsevier Ltd. All rights reserved.

SELF-ASSESSMENT

adiation and an autologous stem cell graft. Prior to this, he r was well; however, he was prescribed a salbutamol inhaler to use before exercise. On examination he was noted to have ne crepitations audible throughout both lung elds and there was good air entry bilaterally. The rest of the clinical examination was normal. His full blood count showed the following result: Haemoglobin White cell count Neutrophils Lymphocytes Platelets 13.1 g/dl 5.8 109/L 45% 40% 214 109/L

C.  Bone marrow aspirate D.  MRI chest E.  Bronchoscopy and lavage 3.  What is the most likely cause of these results? Choose ONE answer ONLY from the following: A.  Asthma B.  Interstitial lung disease C.  Mediastinal mass causing large airway obstruction D.  Bronchiectasis E.  Pneumocystis pneumonia

Answers
Case 1 1.  D 2.  A 3.  B Congenital central hypoventilation syndrome (CCHS) is a rare genetic condition characterized by signicant under breathing, particularly during quiet sleep, and a lack of response to hypoxia and hypercarbia. Unrecognized this condition can be fatal or result in hypoxic brain injury. Infants present with apnoeic episodes or cyanosis, with respiratory studies showing poor respiratory air ow associated with a lack of chest and abdominal wall movement, hypoxia and progressive hypercarbia. CCHS is a lifelong condition, which is associated with multiple abnormalities in the autonomic nervous system, including reduced heart rate variability, reduced ability to produce a febrile response to illness and oesophageal dysmotility, and 1015% have Hirschsprung disease. In 90% of CCHS patients the PHOX2b gene has been identied and genetic studies are widely available. The gene is located on chromosome 3p12 and codes for a homeobox transcription factor. All patients require mechanical ventilatory support whilst sleeping and 35% require long-term 24-h ventilation. In infants this is best delivered as positive pressure mechanical ventilation via tracheostomy, although in some older children, who only require night-time ventilation, non-invasive bi-level pressure ventilation can be used; diaphragmatic pacing has also been used in some patients.

His pulmonary function tests are shown below. Normal range Spirometry Forced expiratory volume in 1.0 s (L) Forced vital capacity (L) FEV1/VC (%) Peak expiratory ow rate (L/s) Plethysmography Slow vital capacity (L) Functional residual capacity (L) Residual volume (L) Total lung capacity (L) Transfer factor (mmol/min kPa) Presalbutamol value 1.10 1.22 90 198 Postsalbutamol value 1.15 1.32 87 205

1.892.92 2.313.44 7792 214394

1.273.44 2.41 0.521.39 2.984.43 4.838.39

1.34 1.66 0.5 2.48 3.1

1.41 1.64 0.49 2.53

1.  Which one of the following options is best demonstrated by the pulmonary function tests shown? Choose ONE answer ONLY from the following: A.  Obstructive airway disease with signicant reversibility B.  Obstructive airway disease with no signicant reversibility C.  Restrictive lung disease Air trapping D.  E.  Poor technique 2.  Which of the following investigations would be most useful in determining the diagnosis? Choose ONE answer ONLY from the following: A.  Histamine challenge B.  High-resolution CT chest

FURTHER REaDING Chen ML, Keens TG. Congenital central hypoventilation syndrome: not just another rare disorder. Paediatr Respir Rev 2004; 5: 1829. Weese-Mayer DE, Berry-Kravis EM. Genetics of congenital central hypoventilation syndrome: lessons from a seemingly orphan disease. Am J Respir Crit Care Med 2004; 170: 1621.

PAEDIATRICS AND CHILD HEALTH 19:4

187

2008 Elsevier Ltd. All rights reserved.

SELF-ASSESSMENT

Case 2 1.  A 2.  E 3.  B Job syndrome (also called the hyper IgE-syndrome) is a rare multisystem disorder of immunity and connective tissue. Children usually have recurrent pneumonias and infected eczema, with Staphylococcus aureus being the most common pathogen, leading to the development of pneumatocoeles in the lung and skin abscesses, which are frequently cold in nature. In addition, there are a number of skeletal symptoms, including hyperextensible joints, scoliosis, retained primary dentition and coarse facial features in older children. Laboratory investigations demonstrate extremely elevated IgE levels (more than 2000 IU/ml) and raised eosinophil counts. Job syndrome has been identied in all ethnic groups and is found equally in both sexes. It may be sporadic in nature and, although most pedigrees are consistent with an autosomal dominant inheritance, this may vary. Recent studies have identied mutations in the signal transducer and activator of the transcription 3 (STAT3) gene. The underlying mechanism of Job syndrome is still not clear but theories include an imbalance in the normal T helper 1 cell (Th-1)/T helper 2 cell (Th-2) cytokine response, leading to increased IgE production. There is no specic therapy available, although aggressive treatment of active infection and antistaphylococcal prophylaxis is important. Immune modulators such as interferon-, cyclosporine A and intravenous immunoglobulin infusions have been used with limited response; bone marrow transplantation has not been shown to be curative.
FURTHER REaDING Grimbacher B, Holland SM, Puck JM. Hyper-IgE syndromes. Immunol Rev 2005; 203: 24450. Holland SM, DeLeo FR, Elloumi HZ, et al. STAT3 mutations in the hyper-IgE syndrome. N Engl J Med 2007; 357: 160819.

in males and the highest incidence is seen between the ages of 1 and 4 years. Seventeen per cent of cases yield a positive organism, with Streptococcus pneumoniae serotype 1 being responsible for 50% of these cases. Blood cultures, sputum cultures and pleural uid should be sent for analysis. Identication yields may be increased by using polymerase chain reaction and latex agglutination techniques. All children with empyema or loculated parapneumonic effusion should be managed in the respiratory unit of a tertiary referral centre. Delay in diagnosis leads to increased morbidity and a prolonged hospital stay, due to the difculties in treating advanced, organized empyemas. Early intervention with chest drain insertion reduces the period of illness and hospital stay. A small bore chest drain should be inserted under ultrasound guidance. There is no evidence that large bore drains are preferable; small bore drains allow for more patient movement and are better tolerated. Intrapleural brinolytics (such as urokinase) have been shown to reduce hospital stay. In some centres a mini-thoracotomy or video-assisted thoracotomy (VATS) are used prior to chest drain insertion. If symptoms continue, a thoracic surgeon should be consulted as decortication may be required. In contrast to adults, the long-term outcome for children with empyema is extremely good, with the majority of children being clinically back to full health by 4 weeks and with most chest radiographs returning to near normal in 36 months. The majority of children who develop empyema will have been in good health prior to the acute infection and will have no long-term clinical consequences; provided the chest X-rays return to normal, further follow up investigations are unnecessary.
FURTHER REaDING Balfour-Lynn IM, Abrahamson E, Cohen G, et al. BTS guidelines for the management of pleural infection in children. Thorax 2005; 60 (Suppl 1): i121. Eastham KM, Freeman R, Kearns AM, et al. Clinical features, aetiology and outcome of empyema in children in the north east of England. Thorax 2004; 59: 5225. Roxburgh CS, Youngson GG, Townend JA, Turner SW. Trends in pneumonia and empyema in Scottish children in the past 25 years. Arch Dis Child 2008; 93: 3168.

Case 3 1.  D 2.  B 3.  C The hyponatraemia was due to syndrome of inappropriate antidiuretic hormone (SIADH). ADH is usually secreted in response to rising plasma osmolality in order to increase reabsorption of water in the distal tubules and the collecting ducts of nephrons. In certain pathological situations, including lower respiratory tract infections, too much ADH is secreted, leading to excess water retention and hyponatraemia, with uid restriction being the most appropriate treatment. The incidence of empyema has been steadily increasing since the mid 1990s and this appears to be unrelated to any increase in cases of pneumonia. Empyema is more common

Case 4 1.  C 2.  B 3.  B Spirometry shows a proportional reduction in both forced expiratory volume in 1 s (FEV1) and forced vital capacity (FVC), with the FEV1/FVC ratio being within normal limits, excluding obstructive airways disease. Plesthsymography demonstrates a reduced total lung capacity and residual volume, which indicates a restrictive lung disease. Transfer

PAEDIATRICS AND CHILD HEALTH 19:4

188

2008 Elsevier Ltd. All rights reserved.

SELF-ASSESSMENT

actor is a measure of how readily carbon monoxide crosses f the interstitium and is reduced if inltrates or brosis are present within the lung parenchyma, as found in interstitial lung disease. High-resolution CT is the most sensitive imaging modality to conrm the diagnosis of interstitial lung disease and demonstrates the extent of parenchymal lung disease; it may also determine suitable biopsy sites should a tissue diagnosis be required. Restrictive lung disease may be secondary to the following causes:  intrinsic lung disease (e.g. primary interstitial lung diseases, infectious and post-infectious disorders)  extrinsic disorders (e.g. scoliosis, neuromuscular disorders such as Duchenne muscular dystrophy, morbid obesity)  secondary to systemic disease (e.g. sarcoidosis, histiocytosis)  drug induced (e.g. chemotherapy agents, radiation). Paediatric interstitial lung disease is rare. The types, causes and prognosis are ill-dened, and do not follow the patterns seen in adults. In approximately half of cases, the aetiology is unknown. In this case, the restrictive defect is most probably a consequence of treatment of the stage IV neuroblastoma. Many of the treatments used to treat childhood cancer can result in pulmonary brosis in subsequent years,

including alkylating agents such as bulsulphan, bleomycin and cyclophosphamide. Radiotherapy can also cause pulmonary brosis, particularly if used in combination with alkylating agents. The prognosis of pulmonary brosis in cancer survivors is variable and symptoms may improve, remain unchanged or worsen with time. Any child at risk of developing restrictive lung disease should have baseline pulmonary function tests performed at the end of therapy and repeated if clinically indicated. The treatment of interstitial lung disease in children is dependent upon the aetiology. Steroids remain the primary therapeutic option, with other immunosuppressive agents such as azathioprine, cyclophosphamide or hydroxycholoroquine being of benet in some patients.

FURTHER REaDING Fan LL, Deterding RR, Langston C. Pediatric interstitial lung disease revisited. Pediatr Pulmonol 2004; 38: 36978. Mertens AC, Yasui Y, Liu Y, et al. Pulmonary complications in survivors of childhood and adolescent cancer, a report from the Childhood Cancer Survivor Study. Cancer 2002; 95: 243141. Scottish Collegiate Guidelines Network. Long term follow up of survivors of childhood cancer. A national clinical guideline, 2004. http://www.sign.ac.uk/pdf/sign76.pdf.

PAEDIATRICS AND CHILD HEALTH 19:4

189

2008 Elsevier Ltd. All rights reserved.

SELF-ASSESSMENT

Self-assessment
Questions
Question 1 A 7-year-old boy presents with a history of right-sided knee pain, fever and increasing tiredness. He had been well until 3 weeks prior to his presentation, when he had an upper respiratory tract infection. Since then, his parents report that he has a reduced appetite and has lost weight. He has missed a signicant number of days of school and has been unable to participate in his usual sports activities as he becomes short of breath. He has a past history of asthma, for which he uses his salbutamol inhaler infrequently. There is no family history of note, no history of foreign travel and his immunisations are all up-to-date. His observations are as follows:
Temperature Pulse Respiratory rate Blood pressure 38.5C 120 beats/minute 32 breaths/minute 110/56mmHg

X-rays of his chest and knee are shown in Figure 1a and b. His blood lm is shown in Figure 1c. (b)  Which of the following is the most likely diagnosis? (Choose ONE) Aplastic anaemia B-cell lymphoblastic lymphoma Cytomegalovirus (CMV) infection Juvenile idiopathic arthritis T-cell acute lymphoblastic leukaemia (c)  The most important management at this stage would be: (Choose THREE) Abdominal ultrasound scan Broad-spectrum intravenous antibiotics Human leukocyte antigen (HLA) tissue-type Lumbar puncture Intravenous dextrose saline with potassium Measure the serum urate level Oral ibuprofen for management of pain and pyrexia Send blood for CMV polymerase chain reaction Start chemotherapy

Clinical examination shows that he is pale and tired but alert. His mucous membranes are not dry, but pale. He is noted to have non-tender cervical lymph nodes bilaterally, measuring between 1 and 2 cm, and shotty axillary and inguinal nodes. His throat reveals enlarged tonsils, although they do not appear to be inamed. His respiratory and cardiovascular examinations are normal, although it is noted that he becomes distressed when lying supine. Abdominal examination reveals a liver edge that is palpable 1 cm below the right costal margin, and a spleen tip just below the left costal margin. Kernigs and Brudzinskis signs are negative. He does not have any skin lesions, bruises or petechiae. He is noted to have a good range of movements in both knees, with no swelling or bony deformity. (a)  Which of the following investigations would be most appropriate at this stage? (Choose THREE) Auto-antibody screen Electrocardiogram (ECG) Ferritin Fine-needle aspiration of node Full blood count Haemoglobin electrophoresis Lumbar puncture Mantoux test Monospot test

Figure 1 a Chest x-ray of 7-year-old boy; b Knee x-ray of the same boy; c Blood lm of the same boy.

Catherine Morley-Jacob MBBS MRCP MRCPCH DCH MD is a Consultant Paediatric Oncologist at the Department of Paediatric Oncology, Childrens Hospital for Wales, Cardiff, UK. Shahera Rahman MBChB MRCPCH is a Specialist Registrar in Paediatric Oncology at the Department of Paediatric Oncology, Childrens Hospital for Wales, Cardiff, UK.

Question 2 A 5-month-old girl is referred by her general practitioner to the assessment unit with pyrexia. Her parents report that she has been spiking temperatures for the past 2 days and is feeding less than usual. She has had a slightly runny nose but no cough. She has been vomiting, although there is no history of

PAEDIATRICS AND CHILD HEALTH 19:5

241

2009 Elsevier Ltd. All rights reserved.

SELF-ASSESSMENT

diarrhoea. There is no history of offensive-smelling urine. She has been previously been t and well, with no past medical history of note. On examination, she has a temperature of 38.5 C. She is sleeping but easily rousable, although grizzly. She has a normal cry and normal movements. She has a full, wet nappy. Systemic examination does not reveal any abnormalities and her observations show:
Heart rate Respiratory rate Capillary rell time 105 beats/minute 40 breaths/minute <2 seconds

just palpable, and macroglossia. Clinically she has features of BeckwithWiedemann syndrome. (a)  What other clinical features would you look for in this baby, now and throughout her childhood? (Choose TWO) Aniridia Cafe-au-lait patches Cardiomyopathy Cleft palate Hemihypertrophy Umbilical hernia (b)  The appropriate management at this stage would be: (Choose ONE) Check her blood glucose level Refer to clinical genetics Request an urgent abdominal ultrasound scan Review her 3-monthly to examine her abdomen Send urine for catecholamines (c)  Which of the following tumours is she most at risk from? (Choose TWO) Ewings sarcoma Hepatoblastoma Neuroblastoma Rhabdomyosarcoma Teratoma Wilms tumour Question 4 A 9-month-old presents with a small, painless, purple lump just below his left clavicle. He is well and thriving. His mother describes him as a happy, lazy baby who is always smiling, who interacts socially and appropriately but who refuses to sit unaided. In the clinic he is sitting propped up on his mothers lap playing happily with her keys. You suspect the diagnosis may be neuroblastoma. He makes no effort to bear weight when held upright. (a)  Which of these topics is important to discuss with his mother? (Choose ONE) Diarrhoea Petechiae Recurrent infections Skin ushing Urinary stream Respiratory and cardiovascular examinations are unremarkable. His abdomen is soft, with no palpable masses or organomegaly. Examination of his legs shows bilateral accid paralysis with no sensation. His chest x-ray is shown in Figure 2. (b)  The appropriate next management step is: (choose ONE) Refer urgently for removal of subcutaneous lump Refer urgently for removal of the mass seen on x-ray Request an urgent magnetic resonance imaging (MRI) scan

(a)  What further tests would be most appropriate at this time: (choose TWO) Capillary blood gas Chest x-ray Full blood count Lumbar puncture Nasopharyngeal aspirate Urine for microscopy (b)  The urine dipstick is negative for leukocyte esterase and positive for nitrites, and the urine is sent for microscopy and culture. The appropriate management at this stage would be: (Choose ONE) Await results of urine culture and if positive, start antibiotics Commence intravenous (i.v.) ceftriaxone Commence 710 days of oral trimethoprim Commence 35 days of oral augmentin Repeat urine and then start antibiotics (c)  The urine culture shows Escherichia coli at more than 100,000 organisms/ml, with typical sensitivities. She has responded well to treatment and is apyrexial within 48 hours of commencing antibiotics. The ongoing management of this patient should include: (Choose ANY number) Dimercapto-succinic acid (DMSA) scan Follow-up in general paediatric clinic Micturating cystourethrogram (MCUG) Prophylactic trimethoprim 6-week ultrasound scan Ultrasound scan during acute episode Question 3 A term baby is seen in the out-patient clinic as follow-up for hypoglycaemia in the neonatal period. She is the second child of Afro-Caribbean parents. The pregnancy was unremarkable and she was born by normal vaginal delivery with Apgars of 7, 9 and 9. She weighed 2.95 kg and had no obvious dysmorphic features at birth. She was admitted to the neonatal unit at 12hours of age and was discharged back to her mother at 3 days of age. She is now 8 weeks old and is well and smiling but her parents are worried that she keeps sticking her tongue out. She is feeding well, is 10% below her birth centile for weight, and is alert and active. On examination she appears well, and the only abnormal ndings are a liver edge,

PAEDIATRICS AND CHILD HEALTH 19:5

242

2009 Elsevier Ltd. All rights reserved.

SELF-ASSESSMENT

Tumour mass behind heart

Figure 2 Chest x-ray of 9-month-old baby.

Request an urgent ultrasound scan Send urine for catecholamines All your investigations are back and his urinary catecholamines are high, conrming your suspected diagnosis. He has the subcutaneous nodule removed and it is sent for histology and biological markers. (c)  Features that rule out a diagnosis of stage 4S neuroblastoma (with its good prognosis) include: (Choose ANY number) Bone marrow involvement Bone metastases High platelet count Liver inltration N-myc amplication Subcutaneous nodules Young age

Answers
Question 1 (a)  Auto-antibody screen Full blood count Monospot test (b)  T-cell acute lymphoblastic leukaemia (c)  Broad-spectrum intravenous antibiotics Lumbar puncture Measure the serum urate level There is potentially a large differential diagnosis for a pyrexial child including infection, malignancy, drug-induced and other chronic inammatory conditions such as juvenile idiopathic arthritis, inammatory bowel disease and Kawasaki disease. The clinical symptoms in this case are vague, apart from the pyrexia, as is often seen in malignancies. Given the

history and clinical ndings, a full blood count, as well as other blood tests including blood cultures, C-reactive protein, erythrocyte sedimentation rate and liver function tests should be performed. As infectious mononucleosis often presents with similarly vague symptoms, a monospot test is also appropriate. An auto-antibody screen, although non-specic, is a good screening tool in autoimmune disease. Although a lumbar puncture should be considered in all patients with pyrexia of unknown cause, the patient in this case does not appear to have symptoms suggestive of meningism. This patient has T-cell leukaemia. The chest x-ray shows mediastinal widening, with left lower zone consolidation and the peripheral blood lm shows blast cells. The knee x-ray is normal. If the blood lm had been normal and lymphoma was suspected, a complete lymph node should be removed as the diagnosis is difcult to make on ne-needle aspiration. There are different types of childhood leukaemia, but the most commonly occurring in children is acute lymphoblastic leukaemia (ALL). Lymphomas, in general, are divided into two broad categories, Hodgkin lymphoma (HL) and non-Hodgkin lymphoma (NHL). The criteria utilised to distinguish between lymphoma and leukaemia has been debated for years. While both can be of B-cell or T-cell phenotype, the distinction is based on the degree of bone marrow involvement. Children who have greater than 25% inltration of their marrow with blast cells are considered to have ALL. T-cell disease is more likely than B-cell to present with a mediastinal mass and a high peripheral white cell count. A high peripheral white cell count indicates bone marrow involvement and therefore leukaemia. Except for the high number of lymphoblasts on the peripheral lm, none of the clinical signs and symptoms is pathognomonic of leukaemia. Other diagnoses to consider include viral infections such as EpsteinBarr virus, cytomegalovirus, other malignancies such as neuroblastoma, haematological disorders such as aplastic anaemia, histiocytosis, idiopathic (immune) thrombocytopenic purpura (ITP) and juvenile idiopathic arthritis. Prior to instituting specic therapy, measures should be instituted to prevent complications, particularly in patients with a

PAEDIATRICS AND CHILD HEALTH 19:5

243

2009 Elsevier Ltd. All rights reserved.

SELF-ASSESSMENT

high white cell count or bulk disease. Serum urate, renal function, serum calcium, phosphate and magnesium should be measured regularly in these patients to monitor for tumour lysis. Tumour lysis can occur spontaneously or as a result of chemotherapy leading to serious metabolic complications such as hyperuricaemia, hyperkalaemia and hyperphosphataemia. This combination can ultimately lead to renal failure or cardiac arrest if left untreated. Prevention is with hyperhydration, anti-urate drugs (allopurinol or rasburicase) and maintenance of urine output. Fluid should not contain potassium, as this may exacerbate hyperkalaemia. Pyrexia in patients may be secondary to the disease process itself or due to sepsis. Given that the immune system is compromised in such patients, despite the high white cell count, they should routinely be given broad spectrum intravenous antibiotics, even with low grade pyrexia. Ibuprofen is not recommended in these patients as poor platelet function in patients with thrombocytopenia may lead to bleeding. Airway compromise and pleural effusions can be an acute problem with large mediastinal masses. The diagnosis of ALL must be established beyond any doubt and, therefore, a complete work-up is generally considered as important as rapidly initiated therapy in a stable child.

Bone marrow aspiration should be performed for morphology, immunophenotyping and cytogenetics and a lumbar puncture to exclude central nervous system disease. A liver ultrasound is unlikely to aid diagnosis or management, as the hepatosplenomegaly is a reection of disease burden. A child with leukaemia would not routinely need a bone marrow transplant and, therefore, routine HLA typing is not necessary at diagnosis. Question 2 (a)  Full blood count Urine for microscopy (b)  Commence 710 days of oral trimethoprim (c)  6-week ultrasound scan According to the National Institute for Clinical Excellence (NICE) guidelines (May 2007), children with a feverish illness should be assessed for the presence or absence of symptoms and signs that can be used to predict the risk of serious illness. For this purpose, a trafc light system was devised, as illustrated in Table 1.

Colour Activity

Green-low risk Normal colour of skin, lips and tongue Responds normally to social cues Content/smiles Stays awake or awakens quickly Strong normal cry/not crying

Amber intermediate risk  Pallor reported by parent/carer Not responding normally to social cues  Wakes only with prolonged stimulation Decreased activity No smile Nasal aring Tachypnoea: RR>50 breaths/mintues, age 612 months RR>40 breaths/minutes, age>12 months  Oxygen saturation95% in air Crackles Dry mucous membranes  Poor feeding in infants Capillary rell time3 seconds Reduced urine output Fever for5 days Swelling of a limb or joint Non-weight bearing/not using an extremity

Red high risk  Pale/mottled/ashen/blue No response to social cues Appears ill to a healthcare professional Does not wake or if roused does not stay awake  Weak, high-pitched or continuous cry  Grunting Tachypnoea: RR>60 breaths/minutes Moderate or severe chest indrawing

Respiratory

Hydration

Normal skin and eyes Moist mucous membranes

Reduced skin turgor

Other

None of the amber or red symptoms or signs

A new lump>2cm

Age 03 months, temperature38C Age 36 months, temperature39C Non-blanching rash  Bulging fontanelle Neck stiffness Status epilepticus Focal neurological signs Focal seizures  Bile-stained vomiting

Table 1 Table modied from NICE Guidance: Feverish illness in children (assessment and initial management in children younger than 5 years May 2007)

PAEDIATRICS AND CHILD HEALTH 19:5

244

2009 Elsevier Ltd. All rights reserved.

SELF-ASSESSMENT

This patient would fall into the intermediate group, with her history of poor feeding and vomiting. NICE guidelines (May 2007) suggest that that urine should be collected and blood tests (full blood count, C-reactive protein and blood cultures) performed. A chest x-ray should also be considered particularly with a temperature more than 39 C and/or a white cell count more than 20 109/l. A lumbar puncture should also be considered, especially in children under the age of 1 year. According to the NICE guidelines for treating urinary tract infection (UTI) in children (August 2007), with a negative leukocyte esterase and a positive nitrite on dipstick, the child should be started on antibiotics. Treatment should not be delayed to collect another urine specimen. If the converse is true, with a positive leukocyte esterase and a negative nitrite, treatment for UTI should only be started if there is good clinical evidence of UTI, as the result may also indicate infection elsewhere. In children with bacteriuria but no systemic symptoms, 3 days of oral antibiotics are sufcient. With systemic symptoms or in children with acute pyelonephritis or upper UTI, a 710 day course of oral antibiotics should be used. If oral antibiotics are not tolerated, i.v. antibiotics should be considered. In children who are septic or in infants under 3 months of age, i.v. antibiotics should be used. Antibiotic prophylaxis should not routinely be used in children following their rst UTI or with asymptomatic bacteriuria, but should be considered with recurrent UTIs. Infants and children with atypical UTIs (seriously ill child, bladder mass, raised creatinine, septicaemia, failure to respond to suitable antibiotics within 48 hours) should have an ultrasound of the urinary tract during the acute infection to identify structural abnormalities of the urinary tract, such as obstruction. In children with their rst UTI that responds to treatment, an ultrasound should only be performed at 6 weeks in infants younger than 6 months, or if the UTI is atypical. DMSA and MCUG scans are required with atypical or recurrent UTIs or with an abnormal ultrasound scan in infants less than 6 months of age. In older children, DMSA scans (only) should be arranged in similar situations. Infants and children who do not undergo imaging investigations should not routinely be followed-up.

Question 3 (a)  Hemihypertrophy Umbilical hernia (b)  Refer to clinical genetics (c)  Hepatoblastoma Wilms tumour BeckwithWiedemann syndrome (BWS) is an imprinting disorder (abnormality at 11p15). Imprinting is a process by which certain genes are switched off during early embryonic development, according to whether they are inherited from the father or the mother. A number of distinct errors involving 11p15 have been identied in BWS patients. Some patients have a rearrangement within the maternal copy of 11p15, while others have paternal uniparental disomy, where the maternal copy of this region is replaced with an extra paternal copy. Affected individuals may have abnormal DNA methylation in different areas of 11p15 or have a mutation of a single gene located within 11p15. Even after extensive molecular testing, the specic defect causing BWS remains unknown in one-third of affected individuals. One of the genes at 11p15, IGF-2, codes for a growth factor protein, explaining the features of overgrowth, including macroglossia and organomegaly. Additional features in BWS include neonatal hypoglycaemia, hemihypertrophy, abdominal wall defects and urogenital abnormalities. Aniridia (absence of the iris) is associated with Wilms tumour but not BWS. Children with clinical features of BWS should be referred to a clinical geneticist for discussion of the genetic implications, to access the diagnostic molecular tests and to start surveillance screening. It is well-known that BWS predisposes an individual to Wilms tumour but there is an increased risk of other embryonal tumours, particularly hepatoblastoma. Abdominal ultrasound scans should pick up both, but alpha-fetoprotein can also be used as a screening tool. The overall risk of developing a tumour in BWS is estimated at 510%. The Wilms tumour does not necessarily occur on the side of overgrowth in hemihypertrophy.

FURtHER REaDING FURtHER REaDING National Institute for Clinical Excellence. Feverish illness in children: assessment and initial management in children younger than 5years. Clinical guideline. London: NICE. May 2007. National Institute for Clinical Excellence. Urinary tract infection: diagnosis, treatment and long-term management of urinary tract infection in children. Clinical guideline. London: NICE. August2007. The Wilms Tumour Surveillance Working Group. Surveillance for Wilms Tumour in at-risk individuals: pragmatic recommendations for best practice. April 2005. Available at: http://www.icr.ac.uk/ research/research_sections/cancer_genetics/cancer_genetics_ teams/genetic_susceptibility/Wilms_tumour_surv_group/2145. shtml. Date accessed: 5/2/09. Choyke PL, Siegel MJ, Craft AW, Green DM, DeBaun MR. Screening for Wilms tumor in children with Beckwith-Wiedemann syndrome or idiopathic hemihypertrophy. Med Pediatr Oncol 1999; 32: 196200.

PAEDIATRICS AND CHILD HEALTH 19:5

245

2009 Elsevier Ltd. All rights reserved.

SELF-ASSESSMENT

Question 4 (a)  Urinary stream (b)  Request an urgent MRI scan (c)  Bone metastases N-myc amplication Stage 4S neuroblastoma often presents in an infant with subcutaneous nodules, a small abdominal primary and liver metastases. However, it can present with a paraspinal mass that extends through a number of spinal foramina into the spinal canal, causing spinal cord compression. This is a medi cal emergency. Signs include a poor urinary stream, urinary dribbling and overow, constipation, sometimes pain and neurological signs below the affected level. Signs of spinal cord compression warrant an urgent spinal MRI to determine whether spinal surgery may be benecial. Surgery should occur as soon as possible because the earlier decompression occurs, the more likely there will be some neurological recovery. Steroids can be used to avert compression while the diagnosis is sought urgently. If the patient has lymphoma, steroids will be effective in the short-term but

the patient may develop tumour lysis syndrome acutely and the lymphoma will recur if not treated further. It is important that a multidisciplinary approach to the management of these patients is taken, both in the short-term and for their long-term rehabilitation. This childs symptoms were longstanding, with a number of months of accid paralysis in his legs, therefore he did not have surgery, but he did require chemotherapy. Figures 3 and 4 show one image from his thoracic MRI and one from his metaiodobenzylguanidine (MIBG) scan. To have stage 4S neuroblastoma, a child must be under 1 year (and possibly 18 months of age) and have a low n-myc level (normal is 2 copies/cell). Signs and symptoms can include bone marrow inltration, liver involvement and skin nodules. Stage 4S disease often resolves spontaneously without the need for treatment; however, it must be watched carefully to ensure it is regressing. Metastatic disease that includes bone metastases and n- myc amplication, especially in an older aged child, will require multimodality treatment, including chemotherapy, radiotherapy, surgery, high dose treatment with autologous stem cell rescue and maintenance therapy.

Collapsed lung and thymus Axillary mass Tumour mass Intraspinal extension Spinal cord Vertebral body

Figure 3 a and b. This gure shows the posterior mediastinal mass with extension through the spinal foramina into the spinal canal.

PAEDIATRICS AND CHILD HEALTH 19:5

246

2009 Elsevier Ltd. All rights reserved.

SELF-ASSESSMENT

b Head Right Arm MIBG uptake in cannula in hand Mass behind heart Normal uptake in liver Bladder (normal)

Figure 4 a and b. This shows the metaiodobenzylguanidine (MIBG) uptake in the mass with normal levels of activity in the liver.

PAEDIATRICS AND CHILD HEALTH 19:5

247

2009 Elsevier Ltd. All rights reserved.

SELF-ASSESSMENT

Self-assessment
Questions
Case 1 A 3-year-old girl presented to the Emergency Department with a 6-week history of a reluctance to walk, and a limp more pronounced in the morning. Prior to onset of these symptoms she had had 2 days of fever with some associated rhinorhoea and cough. A bone scan done at another hospital a week earlier had been reported as normal. There was no recent history of night sweats, weight loss or gastrointestinal symptoms. Treatment had included non-steroidal anti-inammatory drugs (NSAIDs), with minimal improvement. Antibiotics had not been given at any stage. On examination she was afebrile, had an antalgic gait, and on hip examination had limited internal rotation. There was also mild swelling and a decreased range of movement of the left knee. There was no bony tenderness and no obvious swelling or limitation of movement of other joints. There was no lymphadenopathy, and the eye, cardiovascular, respiratory and abdominal examination were normal. (a)  What is the most likely diagnosis? Irritable hip (reactive arthropathy) Juvenile idiopathic arthritis Leukaemia Multifocal osteomyelitis Trauma (b)  What investigation would you recommend? Repeat bone scan Joint aspiration/biopsy Magnetic resonance imaging (MRI) of hips/knee joints with contrast Bone-marrow aspirate (c)  What treatment would you recommend? Corticosteroid joint injections Intravenous methylprednisolone Intravenous antibiotics Surgery Case 2 A 9-week-old boy was well on the day of his routine 2-month scheduled vaccinations (diphtheria/tetanus/pertussis (acellular)/haemophilus inuenzae type b/polio/pneumococcus). He was a little unsettled immediately post-immunization and was put down for a sleep on returning home, now 2 hours postvaccination. When his mother went back into the bedroom 5 minutes later he was staring upwards and she thought he was dead, as he appeared very pale, unresponsive and oppy. She immediately picked him up; he was breathing, and slowly came around over the next 510 minutes. An ambulance was called and he was taken to hospital and monitored for 4 hours, where he remained well. Over the next 24 hours he was more sleepy than normal, but there were no further events. Past medical history: he was born via a normal vaginal delivery. At 1 month of age he had some recurrent vomiting consistent with gastro-oesophageal reux, and settled with thickened feeds. Developmentally he was smiling, interactive, and had normal newborn hearing screening. There is no signicant family history.

Investigations
Full blood count 
Value Haemoglobin Haematocrit RCC MCH MCV RDW Platelets WCC Neut seg Lymphocytes Monocytes Eosinophils Basophils 113g/l 0.32 (L) 4.311012/l 26.2pg 74 L* 12.7% 13.2109/l 5.77109/l 0.63109/l 0.05109/l 0.05109/l 6.69109/l 602109/l (H) Reference range 110140g/l 0.340.42 l 3.95.31012/l 7590 2430pg

1114% 6.017.0109/l 1.58.5109/l 3.09.5109/l 0.11.0109/l 00.810e9/l 00.1109/l 150400109/l

RCC, red cell count; MCH, mean corpuscular haemoglobin; MCV, mean corpuscular volume; RDW, red cell distribution width; WCC, white cell count; L, low; H, high.

Erythrocyte 

sedimentation rate (ESR) 140 mm/h (range 06 mm/h) C-reactive protein (CRP) 63 mg/l (<8 mg/l)  Hip x-ray: normal  Hip ultrasound: thickened synovium bilaterally, nil joint  effusion

Examination
Growth: weight = 5 kg (50th centile)  Head circumference = 41 cm (75th centile)  Length = 58 cm (75th centile) 

Nigel Crawford BMBS MPH FRACP (Paediatrics) is a paediatrician for the NHMRC CCRE in Childhood & Adolescent Immunization, SAEFVICImmunization Safety Surveillance Unit, Department of General Medicine, MCRI/Royal Childrens Hospital, Melbourne, Australia.

On examination he was alert, smiling, had a number of Mongolian blue spots on the buttocks, but no other neurocutaneous stigmata. He had normal tone, power, and reexes. Cardiovascular, respiratory, and abdominal examinations were normal. (a)  What is the most likely diagnosis? Episode related to gastro-oesophageal reux (GORD) Arrhythmia Hypotonic hyporesponsive episode (HHE)

PAEDIATRICS AND CHILD HEALTH 19:6

294

2009 Elsevier Ltd. All rights reserved.

SELF-ASSESSMENT

Atonic seizure Acute life-threatening episode (ALTE) (b)  What investigations would you recommend? Electrocardiogram (ECG) Electroencephalogram (EEG) pH study None required (c)  What would you recommend for future vaccinations No further vaccinations Separate vaccinations (i.e. one vaccine at a time) Vaccinate under supervision in clinic/hospital (d)  Vaccinate as per schedule with normal immunization provider Case 3 A 10-month-old boy presented with a 4-week history of vomiting, increased peripheral oedema and weight loss. Five days prior to admission he developed diarrhoea Six weeks earlier he had had an episode of bronchiolitis, but no other signicant past medical history. He was exclusively breast-fed for 6 months before starting solids, and at presentation was on an extensive diet and three to four breast-feeds daily. There was no recent history of overseas travel, he was not on any medications, and immunizations were up to date.

Value Lymphocytes Monocytes Eosinophils Bands 12.62109/l (H) 2.85109/l (H) 2.44109/l (H) 0.81109/l (H)

Reference range 4.010.0109/l 0.11.0109/l 00.8109/l 00.5109/l

RCC, red cell count; MCH, mean corpuscular haemoglobin; MCV, mean corpuscular volume; RDW, red cell distribution width; Nuc RBC, nucleated red blood cells; WCC, white cell count; I/T ratio, immature to total neutrophil ratio; Neut seg, segmented neutrophils; H, high.

Blood lm
Left shift  High neutrophil count and reactive lymphocytes  Normal morphology  Nil blasts 

ESR: 12 mm/h (range 06) CRP: 18 mg/l (less than 8)

Faeces
Macroscopic appearance: Microscopy Cream unformed White blood cells Red blood cells Fat globules Fatty acid crystals Ethyl acetate conc. performed (Y/N) Parasites Culture Ova, cysts or parasites not detected Salmonella, Shigella, Campylobacter, Aeromonas, Plesiomonas, Yersinia not isolated Not detected Not detected Not detected ++ No growth

On admission
Weight = 8.65 kg (1025th centile)  Blood pressure = 90/60   Pulse rate = 80/min regular Capillary return less than 2 seconds  Oedema of legs (non-pitting) to ankles and mild swelling  Chest clear, normal percussion  Abdomen soft and non-tender, no masses, no organomegaly,  Cardiovascular examination normal  no ascites of hands

Investigations
Full blood count 
Value Haemoglobin Haematocrit RCC MCH MCV RDW Nuc RBC Platelets WCC I/T ratio Neut seg 136g/l (H) 0.40 5.561012/l (H) 24.5pg 72 16.8% (H) 0.4109/l (H) 524109/l (H) 40.7109/l 0.04 14.98109/l (H) (H) Reference range 105135g/L 0.330.41 3.75.31012/l 2331pg 7090 1114% 00 150400109/l 6.018.0109/l less than 0.12 1.08.5109/l Sodium Potassium Chloride Urea Creatinine Total Bili (Bu+Bc) Bu Bc ALP GGT

Electrolytes & liver function tests


Value 130mmol/l (L) 3.2mmol/l (L) 110mmol/l 3.1mmol/l 0.03mmol/l 1mol/l 1mol/l 0mol/l 85IU/l (L) less than 5IU/l 010 mol/l 05mol/l 100350IU/l 040IU/l Reference range 135145mmol/l 3.55.1mmol/l 98110mmol/l 1.36.6mmol/l 0.010.03mmol/l

PAEDIATRICS AND CHILD HEALTH 19:6

295

2009 Elsevier Ltd. All rights reserved.

SELF-ASSESSMENT

Value Total protein Albumin ALT 30g/l (L) 14g/l (L) 54IU/l

Reference range 5071g/l 2945g/l less than 55IU/l

Chest x-ray: normal, no evidence of pleural or pericardial effusion. Ultrasound:  Multiple peristalsing thick-walled uid-lled loops of bowel seen throughout the abdomen No free intraperitoneal uid  No abnormal mass or collection  Normal liver, spleen, kidneys 

Bili, bilirubin; Bu, unconjugated bilirubin; Bc, conjugated bilirubin; ALP, alkaline phosphatase; GGT, -glutamyl transferase ALT, alanine aminotransferase; L, low.

1 antitrypsin
Value Faecal 1 antitrypsin A1 antitrypsin clearance H, high. 1615mg/l (H) 27.8ml/day (H) Reference range 2324mg/l 0.516.4ml/day

(a)  What is the most likely diagnosis?


Intestinal lymphangiectasia Leukaemia Coeliac disease Eosinophilic gastroenteritis Inammatory bowel disease

(b)  What investigations would you recommend? Urine protein/creatinine ratio


Value Urine protein (random) Urine Creatinine: Protein/Creatinine Ratio: 0.11g/l 2.21mmol/l 0.05g/mmol creatinine (less than 0.06) Reference range

Computed tomography (CT) of abdomen Bone-marrow aspirate Serum immunoglobulins Gastroscopy/colonoscopy and biopsy All of the above

(c)  What would you recommend as initial treatment whilst awaiting investigations?
Amino-acid-based formula Medium-chain triglyceride (MCT) formula 20% albumin infusion Gluten-free diet

Coeliac screen
Value IgA (quantitative) Deamidated gliadin peptide Ab IgA Tissue transglutaminase Ab IgA
Ab, antibody; IgA, immunoglobulin A.

Reference range 0.11.29 Negative (less than 20) Negative (less than 20)

0.12 8LU 3LU

Answers
Case 1

(a) Juvenile idiopathic arthritis (JIA) Serum allergy test results


RAST Egg white Milk Wheat 0.02kUA/L 0.05kUA/L 0.06kUA/L

Interpretation Of RAST scores less than 0.35kUA/l: negative 0.350.70kUA/l: low positive 0.73.5kAU/l: moderate positive 3.517.5kAU/l: high positive greater than 17.5kAU/l: very high positive
RAST, radioallergosorbent test.

JIA comprises a group of diseases of unknown aetiology that are manifested by chronic joint inammation. Over the past decade JIA has been treated with more intensive therapy through a multidisciplinary approach to try and minimize the morbidity from chronic arthritis. Multiple classication systems are used internationally, but the key features are onset at less greater than 16 years of age, clinical arthritis, duration of greater than 6 weeks, and type dened by presentation: oligoarthritis/pauci-articular disease, polyarthritis, or systemiconset disease. A thorough rheumatological examination and baseline investigations are very helpful in excluding differential diagnoses, including infection (septic arthritis, osteomyelitis) or postinfectious causes such as reactive arthropathy. The rheumatological differential includes other conditions that can be present with arthritis, including systemic lupus erythematosus,

PAEDIATRICS AND CHILD HEALTH 19:6

296

2009 Elsevier Ltd. All rights reserved.

SELF-ASSESSMENT

juvenile dermatomyositis sarcoidosis and vasculitic syndrome. Arthritis can also be seen in leukaemia, although it is often deeper bone pain, and oligoarthritis can be seen with other chronic diseases such as inammatory bowel disease. Haematological investigations in JIA may show a raised white cell count and inammatory markers. These investigations are often done in conjunction with radiological investigations such as skeletal x-rays, bone scans and ultrasound, especially when considering the differential diagnoses of infection or trauma. Rheumatoid factor is positive in around 710% of older patients with polyarticular JIA and is a indicator of poor prognosis. Other features associated with poor prognsosis include: older age at onset, the presence of rheumatoid nodules, and disease of the cervical spine or hip. JIA rarely presents as isolated arthritis of the hip, so an irritable hip or reactive arthropathy was the most likely diagnosis early in the course of the illness in the case outlined above. JIA became more likely as the symptoms continued at 6 weeks, were worse in the morning, response to NSAIDs was minimal, and other joints became involved.

FurTHer reaDiNG Gardner-Medwin JM, Irwin G, Johnson K. MRI in Juvenile Idiopathic Arthritis and Juvenile Dermatomyositis. Annn y Acad Sci 2009; 1154: 5283. McCann LJ, Wedderburn LR, Hasson N. Juvenile idiopathic arthritis Arch Dis Child Ed Pract 2006; 91: ep29ep36. Miller ML, Cassidy JT. Juvenile rheumatoid arthritis. pp: 1001 1010. In: Kliegman RM, Behrman RE, Jenson HB, Stanton BF, eds. Nelson textbook of pediatrics. 18th Edn. Philadelphia: Saunders; 2007.

Case 2

(a) Hypotonic hyporesponsive episode (HHE)


This reaction post-vaccination is consistent with a signicant adverse event following immunization (AEFI) called a hypotonic hyporesponsive episode (HHE). Other terms for these events have included collapse or shock-like episodes. They have clinical features of the child appearing acutely pale, oppy (hypotonic), and unresponsive. Usually patients do not respond to direct stimulation and slowly come around over the next 130 minutes. They are often more sleepy than usual over the next 24 hours. The reactions can be a frightening experience for parents and physicians. It is an uncommon AEFI which occurs in approximately one in 2040,000 vaccine doses. It is most commonly seen with the rst infant vaccines at 2 months of age, and usually occurs around 34 hours post-immunization, but can occur immediately or up to 2448 hours post-vaccination. It was seen more commonly with whole-cell pertussis vaccine, but does still occur with other vaccines, including those containing acellular pertussis. The differential includes an atonic seizure, but this is usually characterized by unconsciousness rather than hyporesponsiveness, and whilst tone is decreased there is usually no pallor or cyanosis. An acute life-threatening episode (ALTE) has similar features that overlap with an HHE, but classically ALTE has associated apnoea and/or cyanosis. In addition, an HHE, as outlined above, usually occurs within 48 hours of an immunization. In this case there was no history of previous events and no family history or examination ndings to suggest a cardiac anomaly. The pathophysiology of HHE remains uncertain. It appears that multiple factors are implicated, including host or vaccinee characteristics, neurological development, immunological phenomena, and some component of the vaccine(s). The constellation of symptoms described in an HHE is the same as a vaso-vagal or fainting type of event, and this term is preferred in children over the age of 2 years who more commonly have an immediate event post-immunization.

(b) MRI joints with contrast


MRI is a helpful adjunct in investigating joint pathology, as it is the only radiological modality that can differentiate proliferating synovial tissue from an effusion and the other joint components (e.g. cartilage, bone, ligaments and tendons). Since proliferating tissue enhances, contrast (e.g. gadolinium), is required to improve the sensitivity, but as it dissipates quickly only three or four joints can be reviewed at one time. MRI of the joints may also be used to monitor disease progression and response to treatment.

(c) Corticosteroid joint injections


Treatment of JIA aims to minimize longer-term morbidity and progression into adulthood. A multidisciplinary approach includes paediatric rheumatologists, nurse specialists, physiotherapists, occupational therapists and psychologists. Regular ophthalmological review is also required to detect asymptomatic uveitis. Standard drug therapy initially involves the use of NSAIDS and corticosteroid injections into affected joints. Oral or systemic glucocorticoids are usually reserved for overwhelming inammatory or systemic illness (e.g. systemic-onset disease), their longer-term role being limited by side-effects. Methotrexate is the most commonly used second-line agent which can be given weekly either orally or subcutaneously. More intensive therapy is also now offered to refractory patients and may include disease-modifying biologic agents such as etanercept (a tumour necrosis factor (TNF) inhibitor), anakinra (an interleukin 1 (IL-1) receptor antagonist), iniximab (a TFN inhibitor), and adalimumab (an IgG1 humanized monoclonal anti-TNF antibody). Newer drugs are emerging with the ever-increasing understanding of the role of cytokines and cytokine genotypes in subgroups of JIA. This will allow for improved therapy based on individual host factors.

(b) None required


Assessment following an HHE includes a thorough history and physical/neurological examination. Investigations are

PAEDIATRICS AND CHILD HEALTH 19:6

297

2009 Elsevier Ltd. All rights reserved.

SELF-ASSESSMENT

often not indicated, but should be undertaken if the patient has atypical features: e.g. ECG if the cardiac exam is abnormal or there is a family history of arrhythmias, EEG if features or history are suggestive of a possible seizures.

Non-erosive: connective tissue disorders, coeliac diseases,  allergic (eosinophilic) gastroenteritis, parasitic infection

(c) Vaccinate under supervision


In the past an HHE was a contraindication to further whole-cell pertussis immunization, which had an HHE rate of 1 in 3000 vaccine doses, but is not for acellular pertussis or other vaccines. The recurrence rate in the limited number of published studies has been between 0 and 5%. As the recurrence rate is low, most vaccinologists would not recommend separating the future vaccinations required. The difculty is that the HHE episodes can occur up to 24 hours post-vaccination, and many centres would recommend administration of future vaccination under medical supervision either in a day medical unit or with overnight admission, especially if the HHE was delayed. This will often help reassure the parents and ensure the child is adequately protected from vaccine-preventable diseases. Importantly, the studies following up children who have had an HHE showed that these episodes have no long-term effect on development when compared with school age-matched peers.
FurTHer reaDiNG Baraff LJ, Shields WD, Beckwith L, et al. Infants and children with convulsions and hypotonichyporesponsive episodes following diphtheriatetanuspertussis immunization: follow-up evaluation. Pediatrics 1988; 81: 78994. Buettcher M, Heininger U, Braun M, et al. Hypotonic-hyporesponsive episode (HHE) as an adverse event following immunization in early childhood: case denition and guidelines for data collection, analysis, and presentation. Vaccine 2007; 25: 587581. Goodwin H, Nash M, Gold M, et al. Vaccination of children following a previous hypotonic hyporesponsive episode. J Paediatr Child Health 1999; 35: 54952.

Mechanical lymphatic obstruction: For example, intestinal lymphangiectasia, tuberculosis,  sarcoidosis

(b) All of the above


Intestinal lympangiectasia presents with the symptoms of a protein-losing enteropathy, and the obstruction of lymphatic drainage may be due to congenital defects in lymphatic ducts or to secondary causes such as constrictive pericarditis, malrotation, lymphoma and post-cardiac surgery. Patients with congenital lympangiectasia usually present in the rst decade of life, with the rst signs often being peripheral oedema and diarrhoea. Faltering growth may be a presenting feature in younger children. Immunoglobulins are usually low due to protein loss, but opportunistic infections rarely occur. A bone-marrow aspirate should be strongly considered to rule out occult malignancy, which may have supportive examination and haematological ndings. Abdominal ultrasound and CT scans can help identify dilated intestinal loops, thickened folds and nodular protrusions, and can be very helpful when considering other differential diagnoses. Chylous pleural effusions may also be seen on chest x-ray in some cases. Endoscopy and jejunal biopsy is the gold standard in histologic conrmation of intestinal lymphangiectasia. Macroscopically, white nodules and xanthomatous plaques are often visualized and submucosal elevations identied. Because of the patchy nature of the disease in the small intestine, often multiple biopsies need to be taken which show dilated lacteals with distortion of mucosal and submucosal villi in the absence of an inammatory response. Capsule endoscopy is a new technique for visualizing characteristic features in the small intestine which cannot be seen with standard endoscopy.

(c) Medium-chain triglyceride (MCT) formula


Treatment of intestinal lymphangiectasia includes dietary management, minimizing long-chain fat intake by using a medium-chain triglyceride (MCT) formula. Supplementation with fat-soluble vitamins (e.g. A, D, E, and K) is important because of losses due to malabsorption. If an isolated area of bowel is involved, surgical resection may be a management option. Treatment with albumin infusions is usually reserved for haemodynamic instability, as replacement will not treat the underlying cause. Octreotide has been reported as a therapy in refractory cases. Treatment of secondary intestinal lymphangiectasia involves dietary modulation as per above, as well as management of the underlying cause.
FurTHer reaDiNG Sood MR. Disorders of malabsorption. In: Kliegman RM, Behrman RE, Jenson HB, Stanton BF, eds. Nelson textbook of pediatrics. 18th Edn. Philadelphia: Saunders; 2007, pp. 15841593. Radhakrishnan K, Rockson SG. The clinical spectrum of lymphatic disease. Ann NY Acad Sci 2008; 1131: 15584.

Case 3

(a) Intestinal lymphangiectasia


Children with a protein-losing enteropathy classically present with ascites, peripheral oedema, and a low serum albumin. Albumin loss from the gastrointestinal tract is usually less than 15% of the total body albumin, but may be high as 60% in an enteropathy as seen in this case. Faecal antitrypsin levels are elevated as this protein is negligibly broken down in the gastrointestinal tract and is therefore a good screening test for protein loss. There is a long list when It comes to the differential diagnosis of a protein-losing enteropathy. Diagnoses are usually classied into groups depending on the aetiology: Mucosal damage leading to increased permeability: Erosive: ulceration of stomach/duodenum, pseudomem  branous colitis, ulcerative colitis

PAEDIATRICS AND CHILD HEALTH 19:6

298

2009 Elsevier Ltd. All rights reserved.

Das könnte Ihnen auch gefallen